Sunteți pe pagina 1din 42

Atomic Structure

Atom is a Greek word and its meanig Indivisible


e 1.602 1019
i.e. an ultimate particles which cannot be further   1.672 1024  1.672 1027 kg
subdivided. John Dalton considered that “all e / m 9.579 104
matter was composed of smallest invisible Mass of proton in amu =
particle called atom. 1.672  1024
Daltons Atomic Theory :  1.0072amu .
1.66  1024
This theory is based on law of mass
Atomic Models
conservation and law of definite proportions.
A) Thomson’s Model of Atom (1904)
The sailent feature’s of this theory are :-
Thomson was the first to propose a detailed
1) Each element is composed of extremely small
model of the atom.
particles called atoms.
Thomson proposed that an atom consists of a
2) Atoms of a particular element are like but
uniform sphere of positive charge in which the
differ from atom’s of other element.
electrons are distributed more or less uniformly.
3) Atom of each element is an ultimate particle
This model of atom is known as “Pulm-Pudding
and it has a characteristic mass but is
model” or “Raisin Pudding Model” or “Water
structureless
Melon Model”.
4) Atom’s are indestructible i.e. they neither be
Spherical cloud

.in
created nor be destroyed.
5) Atom of element’s take part in chemical of positive charge
reaction to form molecule. + +
al
GOLDEN KEY POINTS + +
Particles carrying negative charge were called + +
rn
negatrons by Thomson. The name negatron was +
+ +
changed to ‘electron’ by Stoney
ou

In cathode ray experiemnt particles (electron) + +


Electron
forming the rays have same specific charge (e/ Thomson's Plum pudding model
m) which is independent of the nature of gas
uj

Drawbacks :
and electrode used. It points out that electrons An important drawback of this model is that
are present in all atoms.
Ed

the mass of the atoms is considered to be evenly


1 spread over that atom.
Mass of electron is times that of proton. It is a static model. It does not reflect the
1837
movement of electron.
rest mass of e  It couldn’t explain the stability of an atom.
e0 
Mass of moving 2 (Where B) Rutherford’s Scattering Experiment
1 v / c 
v is the velocity of the e– and c is the velocity of
light.)
When v  c  mass of e    and
if v  c  mass of e– = imaginary
In anode ray experiment the particles forming
rays have e/m value, dependent on the nature
of the gas taken in the discharge tube, i.e., +ve
Rutherford observed that :
particels are different in different gases.
i) Most of the  -particles (nearly 99.9%) went
Therefore, the mass of the proton can be
straight without suffering any deflection.
calculated.
ii) A few of them got deflected through small
Mass of the proton =
angles.
Atomic Structure
iii) A very few (about one in 20,000) did not nucleon. The magnitude of the +ve charge on
pass through the foil at all but suffered large the nucleus is different for different atoms.
deflections (more than 900) or even came back iii) The volume of the nucleus is very small and
in the direction from which they have come i.e., is only a minute fraction of the total volume of
a deflection of 1800. the atom. Nucleus has a diameter of the order
Following conclusions were drawn from the of 10–12 to 10–13 cm and the atom has a diameter
above observations. of the order of 10–8cm.
i) Since most of the  -particle went straight DA Diameter of the atom 108
  13  105 , DA  105 AN
through the metal foil undeflected. It means that DB Diameter of the nucleus 10
there must be very large empty space within the Thus diameter (size) of the atom is 105 times
atom. the diameter of the nucleus.
ii) Since few of the  -particles were deflected The radius of a nucleus is proportional to the
from their original paths through moderate cube root of the number of nucleons within it.
angles. It was condluded that whole of the +ve
R  A1/3  R  R0 A1/3cm
charge is concentrated and the space occupied
by this positive charge is very small in the atom. Where R0 = 1.33 x 10–13 (a constant) and A = mass
number (p + n) and R = radius of the nucleus.

.in
R  1.33 1013  A1/3cm
iii) There is an empty space around the nucleus
called extra nuclear part. In this part electrons
al
are present. The number of electrons in an atom
is always equal to number of protons present in
rn
the nucleus. As the nuclear part of atom is
responsible for the mass of the atom, the extra
ou

nuclear part is responsible for its volume.


The volume of the atom is about 1015 times the
volume of the nucleus.
uj

When  -particles come closer to the point, they Volume of the atom

10 8 
 1015
Ed

surface force of repulsion and deviate from their Volume of the nucleus 10 13 3
paths. iv) Electrons revolve around the nucleus in
The positively charged heavy mass which closed orbits with high speeds. The centrifugal
occupies only a small volume in an atom is force acting on the revolving e– is being counter
called nucleus. It is supposed to be present at balanced by the force of attraction between the
the centre of the atom. electrons and the nucleus.
iii) A very few of the  -particles suffered strong * This model was similar to the solar system, the
deflections on even returend on their path nucleus representing the sun and revolving
indicating that the nucleus is rigid and  - electrons as planets.
particles recoil due to direct collision with the Drawbacks of Rutherford model -
heavy positively charged mass.
C) Rutherford’s Atomic Model
On the basis of scattering experiments,
Rutherford proposed model of the atom, which
is known as nuclear atomic model. According
to this model.
i) An atom consists of a heavy positively charged i) This theory could not explain stability of atom.
nucleus where all the protons and neutrons are According to Maxwell electron loose its energy
present. Protons & neutrons are collectively continuously in the form of electromagnetic
reffered to as nucleons. Almost whole of the
mass of the atom is contributed by these
Atomic Structure
radiations. As a result of this, the e– should loose
energy at every turn and move closer and closer 6C 12 8 O16
to the nucleus following a spiral path. The p  6 p  8
ultimate result will be that it will fall into the eg. n 0  12  6  6 n0  16  8  8
nucleus, thereby making the atom unstable.
ii) If the electrons loose energy continuously, the e  6 e  8
observed spectrum should be continuous but the Mass no. [A] and atomic weight
actual observed spectrum consists of well defined (a.m.u = atomic mass unit)
lines of definite frequencies. Hence, the loss of Mass of Proton (mp) : 1.673 x 10-27 kg
energy by electron is not continuous in an atom. 1.673 x 10-24 grams
2.2 Atomic Number and Mass Number 1.00750 a.m.u.
a) Atomic Number : [mp mn]
It is represented by Z. The number of protons Mass of Neutron (mn): 1.675 x 10-27 kg
present in the nucleus is called atomic number 1.675 x 10-24 g
of an element. It is also known as nuclear charge. 1.00850 a.m.u.
For neutral atom : [mn>mp]
Number of proton = Number of electron Mass of electron (me) : 9.1 x 10-31 kg

.in
For charged atom : 9.1 x 10-28 g
Number of e– = Z - (charge on atom) 0.000549 a.m.u.
Z= number of protons only Method for Analysis of atomic weight 
al
b) Mass Number (A) Eg. 6C12
It is represented by capital A The sum of number
rn
of Neutrons and protons is called the mass p   6 Weight of Proton = 6 x 1.00750
number of the element; it is also known as n 0  6 Weight of Neutron = 6 x 1.00850
ou

number of nucleons becuase neutron & proton Weight of electron = 6 x 0.000549


are present in nucleus. e 0  6 Weight of Catom = 12.011 a.m.u.
Formula A= number of protons + number of Neutrons
uj

Mass no.of C atom = 12 [p+ and n]


Note : It is always a whole number.
Note : Mass no.of atom is always a whole no.
Ed

Atoms but atomic weight may be in decimal.


Some Important Definitions
a) Isotopes :They are atoms of a given element
Inside the nucleus Outside the nucleus
which have the same atomic number but differ
[n,p'] [e-]
in their mass number.
An atom of the element is represented by Z X A Eg. 6 C 12 , 6C 13 , 6C 14
Where,
8 C 16 , 8C 17 , 8C 18
X = Symbol of element
Z = Atomic number = 1 H 1 , 1 H 2 , 1 H 3
no. of proton = no. of e- (If atom is neutral) Explanation 1 :
A = Mass number = no.of neutron + Atomic no.
6 C 12 6 C 13 6 C 14
11 Na  F 6 C12 p  6 6 6
9 8 O16

eg.
p 
 11 p 
p
 9

 6
p 
 8 e  6 6 6
e  10   e  9  1  10   e  6  e  8
   
n0  6 7 8
[Note : Isotopes have the same nuclear charge
but differ in the number of neutrons in the
nucleus]
Atomic Structure
Explanation 2 : They are the atoms of different element which
1
H1 1
H2 1
H 3
have the same difference of the number of
(Radioactive element) Neutrons & protons.
Protium (H) Deuterium (D) Tritium (T) EX.1 : 5
B11 6
C13
p  1 1 1 p=5 p=6
e=5 e=6
e0  1 1 1 n=6 n=7
n0  0 1 2 n-p=1 n-p=1
15
* Neutron is not available in Protium EX.2 : 7
N 9
F19
* No.of Neucleon = No.of Neutron + No.of Proton p=7 p=9
= n + p+ e=7 e=9
Atomic Weight : The atomic weight of an n=8 n = 10
element is the average of mass of all the isotopes n-p=1 n-p=1
of that element. d) Isotones/Isonutronic Species/Isotonic
An element have three isotopes y1, y2 and y3 They are the atoms of different element which
and their isotopic weights are w1, w2, w3 and have the same number of neutrons.
their percentage/posibility/probability/ratio of EX.1 : 0
H3 2
He4

.in
occurance in nature are x1, x2, x3 respectively, p=1 p=2
then the average atomic weight of eelement is e=2 e=2
n=2 n=2
al
w1 x1  w2 x2  w3 x3 39
Average atomic weight = EX.2 : 19
K 20
Ca40
x1  x2  x3 p = 19 p = 20
rn
Eg : Cl35 Cl37 e = 19 e = 20
Probability 75% 25% n = 20 n = 20
ou

ratio 3 : 1 e) Isoters
35  3  37  1 142 They are the molecules which have the same
Average atomic weight =   35.5
number of atoms & electrons.
3 1 4
uj

b) Isobars EX.1 : CO3 N2O


Given by Aston, Isobars are the atoms of different Atoms = 1+2=3 2+1=3
Ed

element which have the same mass number but Electrons = 6+8x2 7x2+8
different Atomic number i.e they have different = 22 e– =22 e–
number of electrons, protons & neutrons but sum EX.2 : CaO KF
of number of neutrons & protons remains same. Atoms = 2 2
Electrons = 20+8 19+9
EX.1: 1 H 3 2He3 28 e –
28 e–
p=1 p=2 f) Isoelectronic Species
e=1 e=2 They are the atoms, molecules or ions which
n=2 n=1 have the same number of electrons.
p+n=3 p+n=3 EX.1 Cl- Ar
-
18 e 18 e-
EX.2: 19K 40 Ca 40 EX.2 H 2O NH3
20

p=19 p=20 (2+8)=10 e (7+3) = 10 e–
e=19 e=20 EX.3 BF3 SO2

n=21 n=20 (5+9x3)=32 e (16+8x2)=32 e–
p+n=40 p+n=40 GOLDEN KEY POINTS
* Isotopes have same chemical property but
c) Isodiaphers different physical property.
* Isotopes do not have the same value of e/m
Atomic Structure
* Isobars do not have the same chemical & 3
physical property % change = 100  25%
12
* For isotones, A1-Z1=A2-Z2
EX.5: Assuming that atomic weight of C12 si 150
* For isodiaphers, A1-2Z1=A2-2Z2
unit from atomic table, then according to this
EX.1: If the mass of neutrons is doubled & mass
assumption, the weight of O16 will be :-
of electron is halved then find out the new
Sol.  12 amu = 150
atomic mass of 6C12 and the percent by whcih
it is increased. 150
Sol. Step-1 C12  e=6  1 amu =
6 12
p  6  6amu  150
 12amu  16 amu =  16  200 Unit
n  6  6amu  12
If the mass of neutrons is doubled and mass of EX.6: An element have three isotopes and their
e– is halved then. isotopic weight are 11, 12, 13 unit and their
percentage of occurance in nature is 85, 10,
n  12amu  5 respectively then calculate the average
 18amu
p  6amu  atomic weight of element.
Sol. Average Atomic weight =

.in
Note : mass of e– is negligible, so it is not
considered in atomic mass. 11 85  12  10  13  5 935  120  65

Step-2 : %Increment = 85  10  5 100
al
Final mass-Initial mass 18  12
 100   100  50% 1120
Initial mass 12 Average weight =  11.2
rn
100
EX.2: If mass of neutron is doubled, mass of
proton is halved and mass of electron is EX.7: Average atomic weight of an element M is
ou

doubled then find out the new atomic weight 51.7. If two isotopes of M, M52 are present
of 6C12. then calculate the percentage of occurance
of M50 in nature.
uj

12
6C  p  6 Sol. M50 M52
Sol. Step-1 : 12 amu
n  6  x1 + x2 = 100 x2 = (100 - x1)
Ed

If mass of neutron is doubled, mass of proton is average atomic weight =


halved mass of electron is doubled, then new
w1 x1  w2 x2 50  x1  52  x2
n  12   51.7 
atomic mass will be : 15 amu x1  x2 x1  x2
p  3 
Step-2 : %Increment = 50 x1  52 100  x1 
51.7 
Final mass-Initial mass 15  12 x1  100  x1 
 100   100  25%
Initial mass 12 5170 = 50x1 + 5200 - 52x1
EX.3: If no.of protons in X–2 is 16. Then no.of e– 5170 = -2x1 + 5200
in X–2 will be - 2x1 = 30 x1 = 15
1) 14 2) 16 3) 18 4) None M50 = 15% M52 = 85%
Sol.  No.of protons in X–2 is = 16
 No.of electron in X+2 is = 14
EX.4: In C12 atom if mass of e– is doubled and
mass of proton is halved, then calculate the 1. Which of the following statements is
percentage change in mass no.of C12 incorrect for anode rays?
Sol. 6
C12 1) They are deflected by electric and magnetic fields.
e– p+ n0 2) THeir e/m ratio depends on the gas in the
6 6 6 A  12 discharge tube used to produce the anode rays.
12 3 6 A9
Atomic Structure
3) The e/m ratio of anode ray is constant. The radiant Energy do not need any medium
4) They are produced by the ionisation of the for propogation.
gas in the discharge tube. The radiant Energy have electric and magnetic
2. Which of the following pairs have identical fields and travel at right angle to these fields.
value of e/m? The upper most point of the wave is called crest
1) A protons and a neutron and the lower most portion is called through.
2) A proton and deuterium
3) Deuterium and an  -particle Crest Crest

Vibrating source

4) An electron and  -rays a
3. Rutherford’s  -particle scattering a
 Direction

experiments led to the conclusion that of propogation
1) mass and energy are related together Trough Trough
2) the mass and the positive charge of an atom Some of the terms employed in dealing with
are concentrated in the nucleus the waves are described below.
3) neutrons are present in the nucleus 1) Wavelength (  ) (Lambda) : It is defined as
4) atoms are electrically neutral. the distance between two nearest crest or through.
4. The radius of 13Al27 will be 0

.in
1) 1.2 x 10–15m 2) 2.7 x 10–15m It is measured in tems of a A (Angstrom), pm
–15
3) 10.8 x 10 m 4) 4 x 10–15m (Picometre), nm (nanometer), cm(centimetre),
5. Which of the following elements has m(metre)
al
maximum density of nucleus. 0
1) 14Si30 2) 15P31 3) 8O16 1 A  1010 m 1pm = 10–12m,
rn
4) All have same desity 1nm = 10–9 m, 1cm = 10–2m
6. Select iso electronic set 2) Wave number ( v ) (nu bar) : It is the reciprocal
ou

+ +  2 
a) Na , H3O , NH 4 b) CO , NO , H2CO3
3 3 of the wave length that is number of waves
c) P–3, HCl, C2H 5 , PH3 d) F–, Ne, Na+ 1
uj

present in 1cm v
1) a, b, d 2) b, c, d 3) a, b, c 4) a, b, c, d 
It is measured in terms of cm|–1, m|–1 etc.
Ed

7. If the table of atomic masses were established


with the oxygen atom and assigned value of 3) Frequency (v) (nu) : Frequency of a wave is
100, then the mass of carbon atom would be, defined as the number of waves which pass
approximately through a point in 1 s. It is measured terms of
1) 24 2) 75 3) 50 4) 112 Hertz (Hz), s–1, or cycle/s(cps) (1 Hertz = 1s–1)
4) Time period (T) : Time taken by a wave to
1
pass through one point. T= second
v
1) 3 2) 3 3) 2 4) 4 5) 4 6) 4 7) 2 5) Velocity (c) : Velocity of a wave si defined as
distance covered by a wave in 1 second
Electromagnetic Waves (EM Waves) OR
Radiant Energy
c   / T  v or v  c /  or
According to this theory the energy transmitted c  v  s 1     m  or c  v  ms 1 
from one body to another in the form of waves Since c is constants i.e frequency is inversely
and these waves travel in the space with the
propotional to 
same as light (3 x 108 m/s). These waves are
6) Amplitude (a) : The amplitude of a wave is
known as Electro magnetic waves or radiant
defined as the height of crust or depth of
energy. Ex: Radio waves, micro waves, Infra
through.
red rays, visible rays, ultraviolet rays, x-rays,
gama rays and cosmic rays.
Atomic Structure
c  1 Sol. Distance to be travelled from mars of earth = 8
Important note : v   cv  v   x 107 km = 8 x 1010m
  2
Electromagnetic spectrum or EM spectrum :  Velocity of EM waves = 3 x 108 m/s
The arrangement obtained by arranging various Distance 81010 m
 Time =   2.66102 s  4min26s
types of EM waves in orders of their increasing Velocity 3108 m / s
frequency or decreasing wave length is called Planck’s Quantum Theory
as EM SPECTRUM. According to planck’s quantum theory :
( ) 1) The radiant energy emitted or absorbed by a
low(v) RW MW IR Visible U.V Xray  high(v) body not continuously but discontinuously in
low(E) Rays high(E)
longer( ) 3x109A0
7600A0
Shorter(  ) the form of small discrete packets of energy and
150A0
these packets are called quantum.
2) In case of light, the smallest packet of energy
3x1014A0 6x106A0 3800A0 0.1A0 0.01A
0
is called as ‘photon’ but in general case the
smallest packet of energy called as quantum.
EX.8: The vividh Bharti station of All India Radio
3) The energy of each quantum is directly
broadcast on a freqeuncy of 1368 Kilo Hertz
proportional to frequency of the radiation i.e
Calculate the wave length of the Electromagnetic
hc  c

.in
waves emited by the transmitter. E  v  E  h or E   v  
Sol. As we know velocity of light (c) = 3 x 108 m/s   
Give v(frequency) = 1368 kHz = 1368 x 103 Hz h is proportionality constant or Planck’s constant
al
= 1368 x 103 s–1 h = 6.626 x 10–37 kJ s or 6.626 x 10–34 Js or
6.626 x 10–27 erg s
rn
c 3  108 ms 1 4) Total amount of energy transmited from one
     219.3m
v 1368  103 s 1 body to another will be some integral multiple
ou

EX.9: Calculate v in cm –1 and v of yellow of energy of a quantum. E=nhv;


radiations have wavelength of 5800A0 where n is an integer and it is principle quantum
Sol. As we know nhc
uj

number E  nh   nhcv


1 1 10 8 
cm 1
Ed

v 0
 8
 EX.12: Calculate the energy of photon of sodium
5800 A 5800 10 cm 5800
light of wave length 5.862 x 10–16m in Joules.
 17241.37cm 1 Sol.  =5.886 x 10–16m, c = 3 x 108ms–1
v  cv  3 1010 cms 1 1.7 104 cm1 nhc
= 3 x 1.7 x 1014 = 5.1 x 1014s–1 E  nh or  n  1

EX.10: A particular radiostation broadcast at a
frequency of 1120 Kilo Hertz another radio hc 1 6.6  1034 Js  3 108 ms 1
E  
station broadcast a frequency of 98.7 mega  5.862  1016 m
Hertz. What are the wave length of radiations 6.6  3
  10 10 J  3.38  10 10 J
from each station. 5.862
EX.13: Calculate the frequency & energy of a
c 3  108 ms 1 photon of wave length 4000A0.
Sol. Station 1  
st   267.86m
v 1120  103 s 1 Sol. a) Calculation of frequency :
c 3  108 ms 1   4000 A0  4000  1010 m
nd     3.0395m
Station 2
v 98.7 106 s 1 c 3 108 m / s
v    0.75  1015 s 1  7.5 1014 s 1
EX.11: How long would it take a radio wave of  4 107 m
frequency 6 x 103 s–1 to travel from mars to b) Calculation of energy :
the earth, a distance of 8 x 107 km? EX.14: Calculate the  and frequency of a
photon having an energy of 2 electron volt
Atomic Structure
Sol.  1eV = 1.602 x 10–19J  2eV=3.204x10–19J=E The attraction force (Coulombic or electrostatic
a) Calculation of wavelength (  ) : force) between nucleus and electron is equal to
the centrifugal force on electron.
hc hc 6.626 10 34 Js  3  108 ms 1
E or    i.e., Attraction force towards nucleus =
 E 3.204  1019 J centrifugal force away form nucleus
= 6.204 x 10–7m
3rd Postulate :
b) Calculation of frequency (v) :
Electrons can revolve only in those orbits in
c 3 108 ms 1 which angular momentum (mvr) of electron is
v   0.49  1015 s 1  4.9 1014 s 1
 6.204  107 m h
EX.15: Which has a higher energy? integral multiple of
2
a) A photon of violet light with wave length
4000A0 nh h
i.e., mvr   n  
b) A photon of red light with wave length 2 2
7000A0 where : n = +ve Integer number (n=1,2,3,4,...)
Sol. a) Violet light : Eviolet = or (n  I+)
h = Planck’s constant
6.626  1034 Js  3  108 ms 1
10
 4.97  1019 joule  = Constant
4000  10 m

.in
Angular momentum can have values such as
b) Red light : Ered =
h h h h h
hc 6.626 1034 Js  3 108 ms 1 ,2 ,3 ,4 , 5 ... but can not have
 2.8  1019 joule
al
 10
2 2 2 2 2
 7000  10 m
So, Eviolet > Ered h h h
fractional values such as 1.5 ,1.2 , 0.5
rn
...
2 2 2
BOHR’S ATOMIC MODEL 4th Postulate :
ou

Some Important Formulae : The orbits in whcih electron can revolve are
This model was based on quantum theory of known as stationary orbits because in these
radiation and Classical laws of physics. orbits energy of electron is always constant.
uj

Bohr model is applicable only for single electron 5th Postulate :


species like H, He+, Li2+ etc. Each stationary orbit is associated with definite
Ed

Bohr model is based on particle nature of amount of energy therefore these orbits are also
electron. called as energy levels and are numbered as
1,2,3,4,5,.... or K,L,M,N,O,.... from the nucleus
kq1q2
Coulombic force = outwards.
r2 6th Postulate :
mv 2 The emission or absorption of energy in the form
Centrifugal force = of proton can only occur when electron jumps
r
Angular momentum = mvr from one stationary state to another & it is
Important postulates : E  Ehigher  Elower  En2  En1
1st Postulate :
Atom has a nucleus where all protons neutrons = Energy of a quantum
are present. = h = Bohr’s frequency condition
The size of nucleus is very small and it is present Energy is absorbed when electron jumps from
at the centre of the atom. inner ot outer orbit and is emitted when electron
2nd Postulate : moves from outer to inner orbit.
Negatively charged electron revolve around the n2 > n1 whether emission or absorption of energy
nucleus in the same way as the planets revolve will occur.
around the sun.
The path of electron is circular.
Atomic Structure
Shell O E5 (Tangential
Shell N E4 velocity)
Shell M E3
Shell L E2
Shell K r
E1 +Ze e-
Nucleus  + Coulombic
Nucleus force
Shell 1  1q2 
Kq
Shell 2  2 
Shell 3  r 
Shell 4
2
Shell 5 Kq q KZe.e KZe
Columbic force = 12 2   2
r r2 r
9 2 2
Where K = 9 x 10 Nm /coulomb
As we know - Coulombic force = Centrifugal force
1. Electromagnetic radiation travels through
vaccum at a speed of KZe 2 mv 2 2 KZe 2
 or v  ...(1)
1) 186000 m/s 2) 125 m/s r2 r mr
8
3) 3.00 x 10 m/s 4) It depends upon wavelength
nh nh
2. Select incorrect statements. As we know - mvr  or v  ....(2)
2 2 mr

.in
1) Every object emits radiations whose predominant
frequency depends on its temperature. Putting the value of v from eqn.(2) to eqn.(1)
2) The quantum energy of a wave is proportional 2
 nh  KZe 2 n2h2 KZe 2
al
to its frequency.    or 
3) Photons are quanta of light  2 mr  mr 4 2 m 2 r 2 mr
rn
4) The valeu of the planck’s constant depends
n2h2
on energy. r ....  3
4 2 mKZe2
ou

3. What is the wavelength (A0) of a photon that


has an energy of 4.38 x 10–18 J Putting the value of  , h, m, K & e (Constant)
1) 454A0 2) 2.3 x 107A0 in the above eqn. (3)
uj

15 0
3) 6.89 x 10 A 4) 1.45 x 10–15A0
n2
4. A 1kw radio transmitter operates at a r  0.529  108  cm (1A0 = 10–10m = 10–8cm)
Ed

freqeuncy of 800 Hz. How many photos per Z


second does it emit. n2 0
1) 1.71 x 1021 2) 1.88 x 1033 rn  0.529  A
23
Z
3) 6.02 x 10 4) 2.85 x 1030
This formula is only applicable for hydrogen
5. Bohr’s theory is not applicable to
and hydrogen like species i.e. species containing
1) He 2) Li+2 3) He+2 4) H atom
single electron.
B) Velocity of an electron
Since coulombic force = Centrifugal force
1) 3 2) 4 3) 1 4) 2 5) 1 KZe 2 mv 2 2 KZe 2
 or v  ... 1
r2 r mr
Application of Bohr’s Model Putting the value of Angular momentum
A) Radius of Various Orbits (Shell)
nh 2 nh
mvr  or KZe  v
2 2

2 KZe2
v
nh
Putting the value of
Atomic Structure
Z
 , k , e & h v  2.188 106
m/ s
22
 r2  H 0.529 
n   1 4
EX.16: Calculate the radius of 1st, 2nd, 3rd, 4th  r3  Li 2 32 3
0.529 
Bohr’s Orbit of hydrogen. 3
n2 0   r2  H :  r3  Li 2  4 : 3
Sol. Radius of Bohr’s orbit r  0.529  A
Z EX.19: Calculate the radius of 2nd excited state of
12 0
H & 1st excited state of Li+2
st
a) Radius of I orbit : r  0.529  A  0.529 A0 Sol. 2nd excited state, means e– is present in 3rd shell
1
2
b) Radius of IInd orbit :
of hydrogen r3  0.529 
 3
 0.529  9
22 1
r  0.529   0.529  4  2.116 A0 1st excited state, means e– exist in 2nd shell of
1
c) Radius of IIIrd orbit :  2
2
4
+2
2 Li r2  0.529   0.529 
3 3 3
r  0.529   0.529  9  4.761A0
1 nd
radius of 2 excited state of hydrogen
d) radius of IVth orbit :

.in
radius of 1st excited state of Li 2
42 9
r  0.529  0.529  16  8.464 A0 0.529 
1 r  1  27
al
 3 H 
EX.17: Calculate the radius ratio of 3rd & 5th orbit  r2  Li 2
0.529 
4 4
rn
of He+. 3
n2 0 EX.20: Calculate velocity of an electron placed in
Sol.  r  0.529  A and Atomic Number of He=2
ou

Z the third orbit of the hydrogen atom. Also


2
calculate the number of revolutions per second
 3 9 that this electron makes around the nucleus.
uj

 r3  0.529   0.529  and


2 2 Sol. Velocity of electron in 3rd orbit :
Z
Ed

r5  0.529 
 5  0.529 
25 Vn  2.182 10 6  ms 1
n
2 2
1
2
V3  2.182  106  ms 1  7.27 105 ms 1
 3 3
r3 0.529  No.of revolution per second
 2  9
Therefore r5  5
2
25 or r3 : r5  9 : 25 vn vn 7.27  105
0.529    
2 2 r3  n 2 a0  2  3.14  9  0.529  1010
2  
EX.18: Calculate the radius ratio of 2nd orbit of  z 
hydrogen and 3rd orbit of Li+2.  2.43 1014 r. p.s
Sol. Atomci number of H=1, Atomic number of
EX.21: How much time an e– will take for one
Li=3,
complete revolution in 2nd orbit of He+?
22 Sol. time taken =
2nd orbit radius of Hydrogen  r2  H  0.529 
1 4
2  3.14  0.529  1010 m
2 distance 2 r 2
3    3.05 1016 s
3rd orbit radius of Li+2  r3  Li  0.529  velocity v 2
3 2.18106  ms1
2
C) Energy of an electron
Atomic Structure
Let the total energy of an electron be E. It is the
1 1
sum of kinetic and potential Energy. En2  En1  13.6  Z 2  2  2 
i.e., K = K.E + P.E.  n2 n1 
2
 1 2   Kq1q2   P..E   KZe 
E   mv      Shell O E5
2   r   r  Shell N E4
Shell M E3
1 2 K .Ze.  e  1 2 KZe 2 Shell L E2
E  mv   mv  Shell K E1
2 r 2 r
Nucleus  +
 1 2 KZe2 
 KE  mv 
 2 2r  Shell 1
Shell 2
Shell 3
KZe2 KZe 2 KZe 2 Shell 4
E   Shell 5
2r r 2r
Energy level for H atom can be represented as follows :
Putting the value of r from eq. (3)
n=6 or P E6 = -0.38eV
KZe2  4 2 mKZe2 2 2 mK 2 Z 2 e 4 n=5 or O E5 = -0.54eV
En  or E n  
2n 2 h 2 n2 h2 n=4 or N E4 = -0.85eV E5-E4=0.31eV

.in
Putting the value of  , K, e, m, h, we get : n=3 or M E3 = -1.51eV E4-E3=0.66eV
n=2 or L E2 = -3.4eV E3-E2=1.89eV
21.69 1019  Z 2 n=1 or K E1 = -13.6eV E2-E1=10.2eV
En   J / atom or
al
n2 i.e., (E2-E1)>(E3-E2)>(E4-E3)>(E5-E4)...
Important Definitions :-
rn
Z2 i) Ionization energy : Minimum amount of
En  13.6  eV / atom
n2 energy required to liberate an electron from the
ou

This formula is applicable for hydrogen atom ground state of an isolated atom is called as
& hydrogen like species i.e. single electron ionization energy.
species. SInce n can have only integral values, n1  1, n2  
uj

it follows that total energy of the e– is quantised. ii) Separation energy : Minimum energy
The -ve sign indicats that the e– is bonded required to remove an electron from its excited
Ed

towards nucleus. state is called as separation energy.


Some extra points :
n1  2,3, 4,5......n2  
2
KZe 1 iii) Excitation energy : Amount of energy
i) K .E  i.e. K .E 
2r r required to shift an electron from ground state
On increasing radius, K.E. decreases to any excited state is called as excitation energy.
KZe 2 1 n1  1, n2  2, 3, 4,5,....
ii) P.E   i.e. P.E  
r r Note : All these kinds of energy are always positive.
On increasing radius, P.E. increases EX.22: If the total energy of an electron is -1.51
KZe2 1 eV in hydrogen atom then find out K.E., P.E.,
iii) E   i.e. E.   orbit, radius and velocity of the electron in
2r r
that orbit.
On increasing radius, total energy increases.
Conclusion : Sol. i) K.E = -E = 1.51eV
ii) PE = 2 x E = -2 x 1.51 = -2.02eV
P.E     2 KE KE     E P.E  2 E
Z2 12
Energy difference between two energy levels : iii)  E  13.6  eV or  1.51  13.6 
n2 n2
Atomic Structure
13.6 z2
 n2   9  n  3 i.e., 3rd orbit E z , n  EH 
1.51 n2
n2 3 3
iv) r  0.529   0.529   0.529  9  4.761A0
Z 1 if z is same if n is same
1 Ez  EH  z 2
Z 1 En  EH  2
8
v) v  2.188  10  = 2.188 x 108x cm/s n
n 3
= 0.729 x 108 cm/s
EX.23: Calculate the energy of Li+2 ion for 2nd
1. In which of the following is the radius of the
excited state
first orbit minimum?
Z2 1) A Hydrogen atom 2) A tritium atom
Sol. E  13.6  2
 Z  3 and e– exist in 2nd state, 3) Triply ionized beryllium
n
means e– present in 3rd shell i.e. n=3. 4) Double ionized helium
2. The energy needed to excite a hydrogen atom
2
 3  13.6eV / atom from its ground to its third excited state is
E   13.6  1) 12.1 ev 2) 10.2 ev 3) 0.85 ev 4) 12.75 ev

.in
 2
 3 3. The ionisation energy of a hydrogen atom is
EX.24: Calculate the ratio of energies of He+ for 13.6ev. The energy of the ground level in
al
1st & 2nd excited state. doubly ionised lithium is
1) -28.7 ev 2) -54.4 ev 3) -122.4 ev 4) -13.6 ev
rn
Energy of (He  ) 1st Excited state 4. What would be the radius of 2nd excited state
Sol. in Li+2 ion?
Energy of (He ) 2nd Excited state
ou

1) 0.529A0 2) 1.51A0 3) 0.2645A0 4) 0.5299A0


2 5. 2nd separation energy of an electron in H
13.6 
 2 atom
uj


Energy of (He ) 2nd shell

 2

9 1) 27.2 ev 2) 1.57 ev 3) 3.4 ev 4) 13.6 ev
2
Energy of (He  ) 3rd shell  2 4 6. How much energy would be required by an
Ed

13.6  2
 3 electron while moving from ground state to
3rd excited state of He+ ion.
EX.25: The ionization energy for the hydrogen 1) 40.8 ev 2) 10.2 ev 3) 51 ev 4) 48.35 ev
atom is 13.6 eV then the requierd energy in
eV to excite it from the ground state to 1st
excited state
Sol. Ionization energy -13.6 eV i.e. 1) 3 2) 4 3) 3 4) 2 5) 2 6) 3
Energy in ground state = -13.6 eV SPECTRUM
Energy of 1st excited state i.e. 2nd orbit = -3.4 eV When a radiation is passed through a
So, E2-E1 = -3.4 + 13.6 = 10.2 eV spectroscope (prism) for the dispersion of the
GOLDEN KEY POINTS radiation, the pattern (photograph) obtained on
Bohr’s atomic model is applicable only for the screen (photographic plate) is called as
monoelectronic species like H, He+, Li+2, Na10+, spectrum of the given radiation
U91+ etc.
Classification of Spectrum

(1) Emmission (2) Absroption


Hydrogen Spectrum
Atomic Structure
When an electric excitation is applied on
hydrogen atomic gas at Low pressure, a bluish Discovered No.of
Series Regions n2  n 1
light is emitted. When a ray of this light is passed by lines

through a prism, a spectrum of several isolated Lyman Lyman U.V.region n2=2,3,4,..../n 1=1 n2 -1
sharp line is obtained. The wavelength of Visible
Balmer Balmer n2=3,4,5,..../n 1=2 n2 -2
region
various lines show that spectrum lines lie in Infra red
Paschen Paschen n2=4,5,6,..../n 1=3 n2 -3
visible, Ultraviolet and the Infra red region. (I.R.)
These lines are grouped into different series. Brackett Brackett I.R.region n2=5,6,7,..../n 1=4 n2 -4
Pfund Pfund I.R.region n2=6,7,8,..../n 1=5 n2 -5
Far I.R.
Humphery Humphery n2=7,8,9,..../n 1=6 n2 -6
region

.in
al
rn
ou
uj
Ed

Similar words n1 = lower energy series


First line/Starting line/Initial line ( max & min ) Rydberg Formula
In 1890, Rydberg gave a very simplest theoretical
Last line/limiting line/marginal line ( min & max ) Equation for the calculation of the wavelength
First line or any series =  line various lines of hydrogen like spectrum
Second line of any series =  line 1 1 1
Third line of any series =  line v  RZ 2  2  2 
  n1 n2 
Calculation of number of spectral lines
a) Total number of spectral lines = where R = Rydberg constant = 109678 cm–1
= 109700 cm–1=10970000 m–1 = 1.1 x 107 m–1
1  2  ...  n2  n1  
 n2  n1  n2  n1  1
1
2  9.12  10 6 cm  912 A0
R
if n1 = 1 (ground state)
Derivation of Rydberg formula :
 n2  1 n2 n  n  1
Total number of spectral lines =   E  En  En
2 2 2 1
b) Number of spectral lines which falls in a
particular series (n2-n1) 2 2 mK 2 Z 2 e 4  2 2 mK 2 Z 2 e4 
E   
where n2 = higher energy series, n22 h 2  n12 h 2 
Atomic Structure
EX.29: The wave number of 1st line of Balmer
2 2 mK 2 Z 2 e 4 2 2 mK 2 Z 2 e4
  series of hydrogen spectrum is 15200 cm–1.
n12 h 2 n22 h 2 The wave number fo 1st line of Balmer series
of Li+2 spectrum will be?
 hc 
 E  h   Sol. Wave number of 1st line of Balmer series of
  
1  1 1 
hc 2 2
2 mK Z e  1 1  2 4 hydrogen spectrum. v   RZ 2  2  2 
   n1 n2 
 2  2  or
 h2  n1 n2 
 1 1 
1 2 2 mK 2e4 Z 2  1 1  (for H, Z=1) v  R  2  2   15200cm1
  2  2  n1 n2 
 ch3  n1 n2  Wave number of 1st line of Balmer series of Li+2
ion is.
2 2 mK 2 e 4
where is a constant which is equal  1 1 
ch3 v  Z 2  R  2  2   Z  3 for Li 2 
to rydberg constant (R).  n1 n2 
 v  32 15200  9 15200  136800cm1
1 1 1

.in
 RZ 2  2  2  EX.30: Calculate the ratio of maximum  of
  n1 n2  Lyman & Balmer series?
EX.26: Calculate the wavelength of 1st line of
al
1
Balmer series in Hydrogen spectrum. Sol. E  v 

rn
Sol. For first line of Balmer series n1=2, n2=3
Maximum  of Lyman series 1st line of Lyman series
1 2 1 1 9  4  5 
 R 1     R   R  Maximum  of Balmer series 1st line of Balmer series
ou

 4 9  36    36  1 1 1 1 1 5
R
 2 2 32  
Lyman B 5 4 5
36 36 1 36     4 9  36   
      9.12  10 6 cm Balmer 1  1 1  1 1 3 36 3 27
uj

5R 5 R 5 R 2  2
L 1 2  1 4 4
 65.66  106 cm  6566 A0 EX.31: In a hydrogen spectrum if electron moves
Ed

EX.27: Calculate the frequency of the last line of from 7 to 1 orbit by transition in multi steps
the lyman series in hydrogen spectrum. then find out the total number of lines in the
Sol. For last line of Lyman series n1=1, n2=  spectrum.
Sol. Lyman = (n2-1) = 7 - 1 = 6
1  1 1  1  Balmer = (n2-2) = 7 - 2 = 5
 Rz 2  2  2   R   0   R
  n1 n2  1  Paschen = (n2-3) = 7 - 3 = 4
Bracket = (n2-4) = 7 - 4 = 3
c 1 Pfund = (n2-5) = 7 - 5 = 2
   c  c  R  3  1010 cm s 1  109678cm 1
  Humphrey = (n2-6) = 7 - 6 = 1
 3.29  10 s 1 15
Total = 21
EX.28: Calculate wavelength of 3rd line of Bracket Total number of lines can be calculated as follows :
series in hydrogen spectrum. Total number of lines =
Sol. For 3rd line of Bracket series n1=4, n2=7  n2  n1   n2  n1   1  7  1 6  1 42
 1
   21
1 1  1 1  49  16  33 2 2 2
 RZ 2  2  2
 R    R R
   4   7   16 49   16  49  784 EX.32: In a hydrogen spectrum if electron moves
from 6th and 2nd orbit by transition in multi
784 784
Therefore,     912 A0  21667 A0 steps then find out the number of lines in
33R 33 spectrum.
Atomic Structure
Sol. Total number of line = 4+3+2+1=10 4) Bohr’s theory does not explain teh fine
or Total number of lines = structure of the spectral lines. Fine structure of
the spectral line is obtained when spectrum is
 n2  n1   n2  n1   1  6  2  4  1 4  5
   10 viewed by spectroscope of more resolution
2 2 2
power.
EX.33: A certain electronic transition from an 5) Bohr theory does not explain the spilliting of
excited state to Ground state of the Hydrogen spectral lines in the presence of magnetic field
atom in one or more steps gives rise of 5 lines (Zemman’s effect) or electric field (Stark’s
in the ultra violet region of the spectrum. effect)
How many lines does this transition produce
in the Infra red region of the spectrum?
Sol. (Lyman Series) ultra violet region : 5 lines
i.e., e– is coming from 6th to 1st Orbit 1. The line spectra of two elements are not
n2-1=5 or n2=6 indentical because
Infrared region line 1) The elements don’t have the same number
i) Paschen series = (6-3)=3 of neutrons
ii) bracket = (6-4)=2 2) They have different mass numbers
iii) Pfund = (6-5)=1

.in
3) Their outermost electrons are at different
Total Number of lines are = 6 energy levels
EX .34: I n H atom if e– moves, from nth orbit by 4) They have different valencies.
al
transition in multi steps, if there are total 2. In which of the following transition will the
number of lines in spectrum are 10 then find wavelength be minimum.
rn
out the value of n. 1) n=6 to n=4 2) n=4 to n=2
 n2  n1   n2  n1   1 3) n=3 to n=1 4) n=2 to n=1
ou

Sol. Total number of lines = 3. The wavelength of third line of the Balmer
2 series for a H atom is
 n  1 n  1  1 21 100 21R 100 R
uj

So, 10  1) 2) 3) 4)
2 100R 21R 100 21
Ed

or 20   n  1 n  4. When the electron of a hydrogen atom jumps


from the n=4 to the n=1 state, the number of
n 2  n  20  0 spectral lines emitted is
n 2  5n  4n  20  0 1) 15 2) 6 3) 3 4) 4
n  n  5   4  n  5  0

 n  4  n  5  0  n = 5
1) 3 2) 3 3) 2 4) 2
Limitation of the Bohr’s model :
1) Bohr’s theory does not explain the spectrum
Wave, Mechanical Model of an Atom
of multi electron atom.
This model consists of following
2) Why the Angular momentum of the revolving
A) de-Brogle concept (Dual nature of Matter)
nh B) Heisenberg’s Uncertainity principle.
electron in equal to , has not been explained
2 A) The Dual nature of matter (The wave
by Bohr’s theory. nature of electron)
3) Bohr inter related quantum theory of radiation In 1924, a French physicist, Louis de-Broglie
and classical law of physics with out any suggested that if the nature of light is both that
theoritical explanation. of a particle and of a wave, then this dual
behaviour should be true for the matter also.
Atomic Structure
1) The wave nature of light rays and X-rays is proved 3) If the radius of a circular orbit is r, then its
on the basis of their interference and diffraction circumference will be 2 r .
and many facts related to radiations can only be 4) We know that according to Bohr theory,
explained when the beam of light rays is regarded
nh
as composed of energy corpuscles or photons mvr 
whose velocity is 3 x 1010 cm/s. 2
2) According to de-Broglie, the wavelength  of nh
or 2 r  ( mv  p momentum) or
an electron is inversely proportional to its mv
momentum p.
nh   
1 h 2 r     de-Broglie equation 
  or   (Here h=Planck’s constant, p  p 
p p
 2 r  n (where n=total number of waves
p=momentum of electron)
 Momentum (p) = Mass (m) x Velocity (v)
1, 2, 3, 4, 5,...  and  =Wavelength
h nh nh
  5)  2 r  or mvr 
mv mv 2
3) The above relation can be confined as follows  mvr = Angular momentum

.in
by using Einsteins equation, Plancks quantum Thus mvr = Angular momentum, which is a
theory and wave theory of light. h
integral multiple of .
al
Einstein’s equation, E=mc2 where E is energy, 2
m is mass of a body and c is its velocity.
rn
c
 E  h  h  (According to Planck’ss

ou

quantum theory) ....(i)


n=5 n=6
and c   (According to wave theory of light)
uj

c Similarity between de-Broglie waves & Bohr's orbital


 

Ed

But according to Einsteins equation E=mc2 ...(ii) 6) It is clear from the above description that
according to de-Broglie there is similarly
c h between wave theory and Bohr theory.
From equation (i) & (ii) : mc2= h  or mc=
  B) Heisenberg Uncertainity Principle
Bohr’s theory considers an electron as a material
h h
or p  or   p particle. Its position and momentum can be
 determined with accurancy. But, when an
4) It is clear from the above equation that the value electron is considered in the form of wave as
of  decreases on increasing either m or v or suggested by de-Broglie, it is not possible to
both. The wavelength of many fast-moving ascertain simultaneously the exact position and
objects like an aeroplane or a cricket ball, is velocity of the electron more precisely at a given
very low because of their hgih mass. instant since the wave extends throghout a
Bohr’s Theory and de-broglie concept : region of space.
1) According to de-Broglie, the nature of an In 1927, Werner Heisenberg presented a
electron moving the nucleus is like a wave that principle known as Heisenberg uncertainity
flows in circular orbits around the nucleus. principle which states that : “It is impossible to
2) If an electron is regarded as a wave, the measure simultaneously the exact position and
quantum condition as given by Bohr in his exact momentum of a body as small as an
theory is readily fulfilled. electron.”
Atomic Structure
The uncertainity in measurement of position, de-Broglie hypothesis is applicable to macroscopie
 x, and the uncertainity in momentum p or as well as microscopic objects but it has no
physical significance for macroscopic objects.
mv , are related by Heisenberg’s relationship
as : (p=mv, p = mv ) h
Remember  0.527  1034 J sec
4
h h
x.p  or x.mv  or EX.35: The mass of a particle is 1 mg and its
4 4
velocity is 4.5 x 105 cm per second. What
should be the wavelength of this particle if
h p h
x.v  or t  x   h=6.652 x 10–27 erg second.
4 m t 4 1) 1.4722 x 10–24 cm
where h is Planck’s constant. 2) 1.4722 x 10–29 cm
3) 1.4722 x 10–32 cm
h h
F  t  x  E  t  4) 1.4722 x 10–34 cm
4 4 Sol. Given that m=1m=1x10–3g, v=4.5 x 105 cm/s,
i) When x  0, v   h=6.652 x 10–27 erg s.
ii) When v  0, x   . So, if the position is h 6.625 1027 ergs

.in
m  
known quite accurately, i.e., x is very small, mv 1103 g  4.5 104 cm / s
v becomes large and vice-versa.  1.4722  1029 cm
al
GOLDEN KEY POINTS EX.36: Which of the following should be the
de-Broglie wavelength in terms of kinetic wavelength of an electron if its mass is 9.1 x
rn
energy. 10–31 kg and its velocity is 1/10 of that of light
1 2 and the value of h is 6.6252 x 10–34 joule
ou

Kinetic Energy (K.E.) = mv or second?


2
1) 2.446 x 10–7 metre
1 2 2
uj

m  K .E  m v or m 2 v 2  2m K.E 2) 2.246 x 10–9 metre


2 3) 2.246 x 10–11 metre
Ed

or mv  2m K .E 4) 2.246 x 10–13 metre


1
h h Sol. Given that m=9.1 x 10–31kg. v= of velocity
10
But   mv     2m K .E of light
1
 mv  2 m K .E .  or v 
10
 3  108 metre/second i.e. 3 x 107
When a chared particle carrying Q coulomb metre/second,
is accelerated by applying potential h = 6.6252 x 10–34 joule second
difference V then K .E.  Q  v Joule. h 6.6252 1034 J .s 6.6252 1034
  
h h mc 9.11031 kg  3 107 m / s 27.3 1024
But   2mK .E    2mQV = 0.2426 x 10–10 metre = 2.426 x 10–11 metre
EX.37: A ball weight 25 g moves with a velocity
 150 0  12.25 0 of 6.6 x 104 cm/s then find out the de-Broglie
For electron    A   A  associated with it.
 V  V
h 6.6  1034  107 erg
The wave nature of electron was verified Sol.   
experimentally by Davisson and Germer. mv 25  6.6  104 cm / s
 0.04  1031 cm  4  1033 cm
Atomic Structure
EX.38: If the uncertainity in position of a moving required and these identification numbers are
particle is 0 then find out p - called as quantum numbers.
a) Principal quantum number (n)  Shell
h h (Orbit)
Sol. xp  or p  or
4 4x b) Azimuthal quantum number (  )  Sub shell
h c) Magnetic quantum number (m)  Orbital
p  or p   d) Spin quantum number(s)  Spin of e–
4  0
EX.39: Calculate the uncertainity in the position a) Principal Quantum Number (n)
of a particle when the uncertainity in Given By  Bohr
momentum is It represents the name, size and energy of the
a) 1x 10–3 g cm s–1 b) zero shell to which e– belongs
The value of n lies between 1 to 
Sol. a) Given p = 1 x 10 cm s–1,
–3
i.e. n=1,2,3,4.....  corresponding name of
h = 6.62 x 10–27 erg s,  = 3.142 shells are K, L, M, N, O,....
According to uncertainity principle Greater the value of n, greater is the distance
h from the nucleus.
x.p  or
4 n2 0

.in
r  0.529  A
h 1 6.62  1027 1 z
x  .   3  0.527 1034 cm
4 p 4  3.142 10 r1  r2  r3  r4  r5 .....
al
b) When the value of p  0 , the value of x Greater the value of n, greater is the energy of
will be infinity. shell
rn
EX.40: The uncertainity in position and velocity z2
of a particle are 10–10 m and 5.27 x 10–24 ms–1 E  13.6  eV / atom
n2
ou

respectively. Calculate the mass of the


E1<E2<E3<E4....
particle (h=6.625 x 10–34 joule second)
Sol. According to Heisenberg’s uncertainity z
uj

Velocity of electron v=2.18x106 m / s


principle, n
Ed

h h v1>v2>v3....
xmv  or m  The angular momentum of a revolving electron
4 4x.v
nh
6.625  1034 is mvr 
  0.099kg 2
4  3.143  1010  5.27  1024
Where n = Prinical quantum number.
EX.41: Calculate the uncertainity in velocity of a The number of electrons in a particular shell is
cricket ball of mass 150 g if the uncertainity equal to 2n2
in its position is of the order of 1A0 (h=6.6 x b) Azimuthal quantum number/Angular
10–34 kg m2s–1). quantum number /Secondary quantum
h number/ Subsidiary quantum number (  )
Sol. x.mv  or
4 Given by -Sommerfeld
It represents the shape of the subshell and orbital
h 6.6  1034 angular momentum
v  
4x.m 4  3.143  1010  0.150 Value of  between 0 to (n-1)
= 3.499 x 10–24 ms–1 i.e.  = 0,1,2.....(n-1)
Quantum Numbers  = 0(s Subshell)
To obtain complete information about an
 = 1(p Subshell)
electron in an atom 4 identification numbers are
 = 2(d Subshell)
 = 3(f Subshell)
Atomic Structure
Ex. If n=1 then  =0  1s i.e. in n=1 shell, only 2. Select the incorrect statements among the
one subshell’s is present. following.
If n=2 then  =0,1  2s,2p i.e. in n=2 shell, h h
two subshell ‘s’ & ‘p’ are present. 1) x.p  2) x.p 
4 4 m
If n=3 then  =0,1,2  3s,3p,3d i.e. in n=3 shell, h h
three subshell ‘s’, ‘p’ & ‘d’ are present. 3) x.V  4) E.t 
4 m 4
If n=4 then  =0,1,2,3  4s,4p,4d,4f i.e. in n=4
3. If the kinetic energy fo an electron is
shell, four subshell ‘s’, ‘p’, ‘d’ & ‘f’ are present.
increased 4 times, the wavelength of the de-
If the value of n is same then the order of energy
Broglie wave associated with it would become
of the various subshell will be s<p<d<f
1) four times 2) two times
Ex. 4s<4p<4d<4f, 3s<3p<3d, 2s<2p
3) half times 4) one fourth times
If value of  is same but value of n is different 4. Velocity of de-Broglie wave in given by
then the order of energy will be.
Ex. 1s<2s<3s<4s<5s<6s c2 hv mc 2
1) 2) 3) 4) v
3d<4d<5d<6s v mc h
4p<5p<6p 5. The representation of an orbital with n=4

.in
h and  =1 :
The orbital angular momentum =     1 1) 4d 2) 4s 3) 4f 4) 4p
2
h  6. Maximum number of electrons present in M

al
or     1 h  h   [h is called as ‘hash’] shell is :
 2 
1) 8 2) 18 3) 32 4) 10
rn
Orbital angular momentum :
For s subshell=0
ou

h
For p Subshell = 2 or 2h
2 1) 2 2) 2 3) 3 4) 2 5) 4 6) 2
The number of electron in a particular subshell
uj

c) Magnetic Quantum Number/ Orientation


is equal to 2  2  1
Quantum Number(m) :
Ed

for s subshell number of electrons = 2e– Given by linde


for p subshell number of electrons = 6e– * It represents the shape of different orbitals and
for d subshell number of electrons = 10e– the orientation of electron cloud (orbital)
for f subshell number of electrons = 14 e– * Under the influence of magnetic field each
Shape of the subshell : subshell is further subdivided into orbitals (The
s  spherical electron cloud is known as orbital)
p  dumb bell shape Magnetic quantum number describe these
d  double dumb bell shape different distribution of electron cloud.
f  complex shape * Value of m=all integral value from -  to + 
including zero.
i.e., Value of m = -  to + 
Orbital : 3D space around the nucleus in which
1. de-Broglie wavelength is related to applied
probability of finding electron is maximum is
voltage is
called an orbital. An orbital can be represented
12.3 0 12.3 0 by 3 set of quantum numbers
1)    A 2)   A
h v   n , ,m (classical representation) = n   n , ,m x*
12.3 0 12.3 0 EX.1: 2px;n=2,  =1, m=-1 or m+1
3)   A 4)   A
EX.2: 3d z 2 ;n=3,  =2, m=0
r m
Atomic Structure
ii) The size and energy of the s-orbital increases
EX.3:  3,2,0 ;n=3,  =2, m=0; 3d z 2
as the principal quantum number increases, i.e.,
Note : It is point/line/plane/surface in whcih the size and energy of s-orbital increases in the
probability of fiding electron is zero order 1s<2s<3s....
y
Total numbers of nodes = n  1 Nodal
surface
They are of 2 types
i) Radial nodes/spherical nodes
number of radial nodes = n- -1 x
ii) Angular nodes/number of nodal planes
z 1s
number of angular nodes/nodal planes= 2s
3s
Nucleus and  are not considered as node. The s orbital of higher energy levels are also
Types of orbitals : symmetrically spherical and can be represented
Case-I : If   0 then m=0 it implies that a as above
subshall has only one orbital called as s orbital.
-1 0 +1
Shapes of s-orbitals : Case-II If   1 p  subshell  then m=
px p z p y
The s-orbitals are spherically symmetrical about

.in
It implies that, p subshell have three orbitals
the nucleus, i.e., the probability of finding s
called as px, py and pz.
electron is same in all directions from the
Shapes of p-orbitals :
al
nucleus. The size of the orbital depends on the
There are three p-orbitals, commonly referred
value of principle quantum number, there is one
rn
to as px, py and pz. These three p-orbitals, posses
spherically symmetrical orbital. The 1s orbital
equivalent energy and therefore, have same
is smaller than 2s-orbital and 2s-orbital is
relation with the nucleus. They, however, differ
ou

smaller than 3s, but all are spherical in shape as


in their direction & distribution of the charge.
shown in figure.
uj
Ed

1S
2S Node These three p-orbitals are situated at right angle
3S Node
Although the s-orbitals belonging to different to one another and are directed along x, y and z
shells are spherically symmetrical, yet they axis (figure)
differ in certain respects as explained below : Each p orbital has dumb bell shape (2 lobes
i) The probability of 1s electron is found to be which are separated from each other by a point
maximum near the nucleus and decreases as the of zero probability called nodal point or node
distance from the the nucleus increases. In case or nucleus).
of 2s electrons, the probability is again The two lobes of each orbital are separated by a
maximum near the nucleus and then decreases plane of zero electron density called nodal plane.
to zero as the distance from teh nucleus Each p orbital of higher energy level are also
increases. The intermediate region (a spherical dumb bell shape but they have nodal surface.
shell) where the probability is zero is called a Nodal surface :
nodal surface of simply node. Thus, 2s-orbital Orbital Nodal Surface
differs from 1s-orbital in having one node within 3px 1
it. Similarly, 3s has two nodes. in general, any 4px 2
ns orbital has (n-1) nodes. npx (n-2)
Atomic Structure
Nodal Plane :
Orbital Nodal plane z z
px yz plane x x
py xy plane
y y
pz xy plane dxy z dyz

y x
Nodal
surface z y z
dz 2
x x
x y y
2Px dzx dx2- 2
y
z 3Px
4Px
Each d-orbital of higher energy level are also
Nodal point double dumbled shape but they have nodal surface.
In d-orbitals :
y y YZ plane y XY plane
i) Nodal Point  1
(Nodal plane) (Nodal plane)
ii) Nodal Surface  3dxy  0 Nodal surface

.in
XZ plane
(Nodal plane) 4dxy  1 Nodal surface
x x x
4dxy  2 Nodal surface
al
z Py z Px z Pz ndxy  (n-3)
Number of nodal surface = n    1
rn
Case-III
When  =2, ‘m’ has five values -2, -1, 0, +1, Nodal y
ou

+2. It implies that d subshell of any energy shell surface


has five orbitals. All the five orbitals are not
identical in shape. Four of the d-orbitals
uj

d xy , d yz , d xz , d x2  y 2 contain four lobes while fifth


x
3dxy
Ed

orbital d z 2 consists of only two lobes. The lobes z 4dxy


dxy orbital lie between x and y axes. Similar is 5dxy
the case for d y 2 and dxz. Four lobes of d x2  y 2
orbital are lying along x and y axes while the iii) Nodal plane : dxy  xy & yz nodal plane :
dxz  xy & zy nodal plane :
two lobes of d z 2 orbital are lying along z axes dzy  zx & yx nodal plane :
and contain a ring of negative charge
d x2  y 2  2, nodal plane :
surrounding the nucleus in xy plane. Geometry
of d orbital is Double Dumb bell d z 2  0, nodal plane :
Note : Orbitals of d subshell are Equivalent in
energy.
d) Spin Quantum number (s) :
m= -2 -1 0 +1 +2
d xy d y 2 d x2 d zx d x2  y 2 Given by Goudsmit and Uhlenback
* It represents the direction of electron Spin
Shape of d-orbitals : around its own axis
It implies that d subshell has 5 orbitals i.e. five * for clock wise spin/spin up(  ) electron
electron cloud and can be represented as follows.
1
2
Atomic Structure
for anitclock wise spin/spin down(  ) electorn Principle : The subshell with minimum energy
is filled up first when this subshell obtained
1 maximum quota of electrons then the next
2
Spin angular momentum of an subshell of higher energy starts filling.
The sequence in whcih various subshell are
h filled are as follows.
e   s  s  1 or s  s  1 h
2
Each orbital can accomodate 2 electrons with n=1 1s
opposite spin or spin paired.
Correct  Spin paired e– n=2 2s 2p

Wrong  Spin parallel e–


n=3 3s 3p 3d
EX.42: For 7py, calculate the value of n,  , m and s
1 1
Sol. n=7,  =1, m=+1 or -1, s=+ or - n=4 4s 4p 4d 4f
2 2
EX.43: For 3s, calculate the value of n,  , m and s.
n=5 5s 5p 5d 5f

.in
1 1
Sol. n=3,  =0, m=0, s=+ or -
2 2
n=6 6s 6p 6d
al
2
EX.44: For 4dz , calculate the value of n,  , m
and s
rn
n=7 7s 7p
1 1
Sol. n=5,  =2, m=0, s=+ or -
2 2
ou

EX.45: Which of the following set of Quantum n = 8 8s


1s2, 2s2, 2p6, 3s2, 3p6, 4s2, 3d10, 4p6, 5s2, 4d10,
number is not possible?
5p6, 6s2, 4f14, 5d10, 6p6, 7s2, 5f14, 6d10,....
uj

1 For example
a) n=2,  =0, m-1, s=-
2 H  1s1
Ed

1
He  1s2
1 2
b) n=3,  =2, m=0, s=  3
Li  1s2,2s1
2 Be  1s2,2s2
4
1 5
B  1s2,2s2,2p1
c) n=2,  =3, m=-2, s=  C  1s2,2s2,2p2
2 6

Sol. a) not possible b) possible c) not possible 7


N  1s2,2s2,2p3
Rules for filling of electrons 8
O  1s2,2s2,2p4
a) Aufbau Principle 9
F  1s2,2s2,2p5
b) (n+  ) rule 10
Ne  1s2,2s2,2p6
c) Hund’s maximum multiplicity principle 11
Na  1s2,2s2,2p6,3s1
d) Pauli’s exclusion principle 12
Mg  1s2,2s2,2p6,3s2
a) Aufbau Principle 13
Al  1s2,2s2,2p6,3s2,3p1
Aufbau is German word and its meaning 14
Si  1s2,2s2,2p6,3s2,3p2
‘Building up’ 15
p  1s2,2s2,2p6,3s2,3p3
Aufbau principle gives a sequence in which 16
S  1s2,2s2,2p6,3s2,3p4
various subshell are filled up depending on the 17
Cl  1s2,2s2,2p6,3s2,3p5
relative order of the Energies of various 18
Ar  1s2,2s2,2p6,3s2,3p6
subshell. 19
K  1s2,2s2,2p6,3s2,3p6,4s1
20
Ca  1s2,2s2,2p6,3s2,3p6,4s2
Atomic Structure
21
Se  1s2,2s2,2p6,3s2,3p6,4s2,3d1
22
Ti  1s2,2s2,2p6,3s2,3p6,4s2,3d2
23
V  1s2,2s2,2p6,3s2,3p6,4s2,3d3
24
Cr  1s2,2s2,2p6,3s2,3p6,4s1,3d5[Exception]
25
Mn  1s2,2s2,2p6,3s2,3p6,4s2,3d5
26
Fe  1s2,2s2,2p6,3s2,3p6,4s2,3d6
27
Co  1s2,2s2,2p6,3s2,3p6,4s2,3d7
28
Ni  1s2,2s2,2p6,3s2,3p6,4s2,3d8
29
Cu  1s2,2s2,2p6,3s2,3p6,4s1,3d10[Exception]
30
Zn  1s2,2s2,2p6,3s2,3p6,4s2,3d10
Electronic configuration can be written by
following different methods :
* 26
Fe  (1) 1s2, 2s2, 2p6, 3s2, 3p6, 4s2, 3d6
(2) 1s2, 2s2, 2p6, 3s2, 3p6, 3d6, 4s2
(3) 1s2, 2s2p6, 3s2p6d6, 4s2 Order : 1s2, 2s2, 2p6, 3s2, 3p6, 4s2, 3d10, 4p6, 5s2,
2 8 14 2 4d10, 5p6, 6s2, 4f14, 5d10, 6p6, 7s2, 5f14, 6d10,...
(4) [Ar] 4s 2
3d 6 c) Hund’s Maximum Multiplicity Rule

.in
(Multiplicity : Many of the same kind)
1s 2 2s 2 2 p 6 3s 2 3 p 6 3d 6 4s 2 * According to Hund’s rule electrons are
* 26
Fe  n
 n  2  n 1
distributed among the orbitals of subshell in
al
n  Outer most Shell or Ultimate Shell or such a way as to give maximum number of
Valence Shell unpaired electron with parallel spin.
rn
In this Shell e– are called as Valance electron or * Thus the Orbital available in the subshell are
this is called core charge first filled singly with parallel spin electron
ou

(n-1)  Penultimate Shell or core or pre before they begin to pair this means that pairing
valence Shell of electron occurs with the introduction of
(n-2)  Pre Penultimate Shell second electron in ‘s’ subshell, fourth electron
uj

* If we remove the last n Shell (ultimate Shell) in ‘p’ subshell, 6th electron in ‘d’ Subshell & 8th
then the remaining shells are collectively called e– in ‘f’ subshell.
Ed

as Kernel. Ex :
Fe  1s 2 2s 2 2 p 6 3s 2 3d 6 3 p 6 4 s 2 5
B 1s2 2s2 2p1
Ex. 26
Kernel
  
b) (n+  ) Rule (For multi electron species)
C 1s2 2s2 2p2
According to it the sequence in which various 6

subshell are filled up can also be determined   


with the help of (n+  ) value for a given
7
N 1s2 2s2 2p3
subshell.
Principle of (n+  ) rule :   
The subshell with lowest (n+  ) value is filled O 1s2 2s2 2p4
8
up first, When two or more subshell have same
(n+  ) value then the subshell with lowest value     
of n is filled up first. 9
F 1s2 2s2 2p5
In case of H-atom :
Energy only depends on principle quantum     
number Ne  1s2 2s2 2p6
10
1s<2s=2p<3s=3p=3d<4s=4p=4d=4f<......
    
Atomic Structure
d) Pauli’s Exclusion Principle EX.2
In 1925 Pauli stated that no two electron in an 29
Cu  1s2 2s2 2p6 3s2 3p6 4s2 3d9
atom can have same values of all four quantum [Wrong configuration]
numbers i.e., An orbital can accomodates
maximum 2 electrons with opposite spin.      
EX.1 : 29
Cr  1s2 2s2 2p6 3s2 3p6 4s1 3d10
C12  1s2 2s2 2p2 [Right configuration]
6

        
n 1 2 2 EX.46: Calculate the number of unpaired e– in
 0 0 1 Cr
m 0 0 +1, 0, -1 Sol. 24Cr  1s2 2s2 2p6 3s2 3p6 4s1 3d5
in 24Cr, 6e– s are unpaired.
s  12 ,  12  12 ,  12  12 ,  12
EX.47: The number of unpaired e– in Cr+3
EX.2 : Sol. Cr+3  1s2 2s2 2p6 3s2 3p6 4s0 3d3
17
Cl  1s2 2s2 2p6 in Cr+3, 3e– s are unpaired.
EX.48: The number of unpaired e– in 3d subshell
  

.in
 
of Cr+3
n= 1 2 2 Sol. 3
= 0 0 1 EX.49: The number of unpaired e– in Fe+2 & Fe+3
al
m= 0 0 +1,-1,0 Sol. Fe+2  1s2 2s2 2p6 3s2 3p6 4s0 3d6 = 4 unpaired e–
rn
1 1 1 1 1 1 1 1 1 1 Fe+3  1s2 2s2 2p6 3s2 3p6 4s0 3d5 = 5 unpaired e–
 ,  ,  , , , ,  ,
2 2 2 2 2 2 2 2 2 2
ou

3s2 3p5

    1. A neutral atom of an element has 2K, 8L,


uj

11M and 2N electrons. The number of p-


n= 3 3
electorn of p-electron in the atom are
= 0 1
Ed

1) 2 2) 12 3) 10 4) 6
m= 0 +1,-1,0 2. An atom has 2 electrons in K-shell, 8
1 1
 ,
1 1 1 1 1
 , , , , electrons in L-shell & 8 electrons in M-shell.
2 2 2 2 2 2 2 The number of p-electrons presents in the
Exception of Aufbau principle : element is :-
In some cases it is seen that the electronic 1) 10 2) 7 3) 12 4) 4
configuration is slightly different from the 3. The maximum number of such electrons in
arrangement given by Afbau principle. A simple an atom with quantum number n=3, l=2 is
region behind this is that half filled & full filled 1) 2 2) 6 3) 10 4) 14
subshell have got extra stability. 4. The number of orbitals in n=3 are
EX.1 1) 1 2) 4 3) 9 4) 16
24
Cr  1s2 2s2 2p6 3s2 3p6 4s2 3d4 5. In the potassium the probable order of
[Wrong configuration] energy level for 19th electron is
1) 3s>3d 2) 4s>3d 3) 4s>4p 4) 4s=3d
 
Cr  1s2 2s2 2p6 3s2 3p6 4s2
24
3d4
[Right configuration]
1) 2 2) 3 3) 3 4) 3 5) 2
 
Atomic Structure
8. (i) 26 Fe54 , 26 Fe56 , 26 Fe57 ,26 Fe58 (a) Isotopes
(ii) 1 H 3 , 2 He3 (b) Isotones
INTRODUCTION (iii) 32 Ge76 , 33As 77 (c) Isodiaphers
1. Rutherford’s  -particle scattering
experiment proved that atom has (iv) 92 U 235 , 90Th 231 (d) Isobars
A) Electrons B) Neutrons (v) 1 H1 , 1D 2 , 1T 3
C) Nucleus D) Orbitals
Match the above correct terms
2. A and B are two elements which have same
A) [(i),-a], [(ii) - d], [(iii)-b], [(iv) - c], [(v) - a]
atomic weight and are having atomic number
B) [(i) - a] [(ii)-d],[(iii)-d], ((iv)-c][v-a]
27 and 30 respectively. If the atomic weight
C) [v -a] [(iv)-c]. [(iii)-d] [(ii)-b] [(i) -a]
of A is 57 then number of neutron in B is
D) None of them
A) 27 B) 33 C) 30 D) 40
9. Choose the false statement about deuterium
3. Find out the nucleus which are isoneutronic
A) It is an isotope of hydrogen
14
A) 6 C, 15 17
7 N, 9 F B) 12
6 C, 14 19
7 N, 9 F B) It contains ((1 e–) + (1 p+) + (1 (n)]
C) It contains only [(1 (p+) + (1 (n)]
C) 14 14 17
6 C, 7 N, 9 F D) 14 14 19
6 C, 7 N, 9 F

.in
D) D2O is called as heavy water
4. Species which are isoelectronic to one 10. If the table of atomic masses is established
another are with the oxygen atom and assigned value of
al
(a) CN– (b) OH– (c) CH 3 (d) N2 200, then the mass of carbon atom would be,
approximately
rn
(e) CO A)24 B)150 C)50 D)112
Correct answer is 11. The relative abundance of two rubidium
ou

A) a, b, c B) a,c,d C) a, d, e D) b,c,d isotopes of atomic weights 85 and 87 are 75%


5. For any anion X–3, the mass number is 14. If and 25% respectively. The average atomic
anion has 10 electrons, then number of weight of rubidium is
uj

neutrons in X2 nucleus A) 75.5 B) 85.5 C) 86.5 D) 87.5


A) 10 B) 14 C) 7 D) 5 12. The ratio of specific charge of a proton and
Ed

6. Which of the following pairs is correctly an  -particle is


matched A) 2 : 1 B) 1 : 2 C) 1 : 4 D) 1 : 1
27
13. I n an atom 13Al , number of proton is (a)
40 40
A) Isotopes 20 Ca, 19 K electron is (b) and neutron is (c). Hence ratio
B) Isotones 30 31
Si, 15 32
P, 16 S will be [in order c : b : a]
14
A) 13 : 14 : 13 B) 13 : 13 : 14
16
C) Isobars 8 O, 17 18
8 O, 8 O C) 14 : 13 : 13 D) 14 : 13 : 14
14. Atomic weight of Ne is 20.2. Ne is mixture of
D) Isoelectronic N 3 ,O2 , Cr 3
Ne20 and Ne22, relative abundance of heavier
7. The atom A, B, C have the configuration isotope is
A  [Z(90) + n(146)], B  [Z(92) + n(146)], A) 90 B) 20 C) 40 D) 10
C  [Z(90) + n(148)] So that 15. Number of protons, neutrons & electrons in
(a) A and C - Isotones (b) A and C - Isotopes
(c) A and B - Isobars (d) B and C - Isobars the element 89  231 is
(e) B and C - Isotopes A)89,231,89 B)89,89,242
The wrong statements are C)89, 142,89 D)89, 71,89
13 17
A) a, b only B) c, d, e only 16. Atoms 6C and 8O are related to each other as
A) Isotones B) Isoelectronic
C) a, c, d only D) a, c, e only
C) Isodiaphers D) Isosters
Atomic Structure
17. The e/m ratio is maximum for D) Does not depend upon mass of electron
A)D+ B) He+ C)H+ D) He2+ 26. The radius of a shell for H-atom is 4.761Ao.
18. Let mass of electron is half, mass of proton The value of n is
is two times and mass of neutron is three A) 3 B) 9 C) 5 D) 4
fourth of orignal masses, then new atomic 27. In Bohr’s atomic model radius of Ist orbit of
weight of O 16 atom Hydrogen is 0.053 nm then radius of 3rd orbit
A) increases by 37.5 %B) remain constant A) 0.159 B) 0.053 C) 0.023 D) 0.026
C) increases by 12.5 %D) decreases by 25 % 28. The first three radius ratio of Bohr orbits
19. An isotone of 32Ge76 is A) 1:0.5: 05 B) 1 : 2 : 3
(i) 32Ge77 (ii) 33As77 C) 1 : 4 : 9 D) 1 : 8 : 27
77
(iii) 34Se (iv) 34Se78 29. For Li+2 ion, r2 : r5 will be
A) (ii) & (iii) B) (i) & (ii) A) 9 : 25 B) 4 : 25 C) 25 : 4 D) 25 : 9
C) (ii) & (iv) D) (ii) & (iii) & (iv) 30. The ratio of the radii of two Bohr orbits of
14
20. In 7N if mass attributed to electrons were H-atoms is 4:1, what would be their
doubled & the mass attributed to protons nomenclature
were halved, the atomic mass would become A) K & L B) L & K
approximately C) N & L D) 2 & 3 both

.in
A) Halved B) Doubled 31. The velocity of electron in third excited state
C) Reduced by 25% D) Remain same of Be3+ ion will be
21. The value of planck’s constant is 3 3
al
A) (2.188 108 )ms 1 B) (2.188 10 6 )ms 1
6.63  1034 Js. The velocity of light is 4 4
rn
–1
3.0  108 ms . Which value of light with C) (2.188  106 )Kms1 D) (2.188  103 )Kms 1
frequency of 8  1015 s 1 . 32. The Bohr orbit radius for the hydrogen atom
ou

A) 3  10 7 B) 2  10–25 (n = 1) is approximately 0.530 A. The radius


–18
C) 5  10 D) 3.75  10–8 for the first excited state (n = 2) will be
22. If change in energy (E)  3  108 J, h = 6.64 A)0.13 A B) 1.0 A C)4.77 A D)2.12A
uj

33. According to Bohr theory, the radius (r) and


1034 J-s and c  3  108 m / s , then velocity (v) of an electron vary with the
Ed

wavelength of the light is increasing principal quantum number ‘n’ as.


A) 6.64  l03Ao B) 6.64  l05Ao A) r increase , v decreases
C) 6.64  10 A–8 o
D) 6.64  1018Ao B) r and v both increases
BOHR’S ATOMIC MODEL C) r & v both decreases
23. Angular momentum in second Bohr orbit of D) r decreases, v increases
H - atom is x. Then find out angular 34. The ratio of radius of first orbit in hydrogen
momentum in 1st excitetd state of Li+2 ion to the radius of first orbit in deuterium will
x be
A) 3x B)9x C) D) x A)1:1 B)1:2 C)2:1 D)4:1
2
35. For any H like system, the ratio of velocities
24. Angular momentum for P-shell electron is of I, II & III orbit i.e., V1 : V2 : V3 will be
3h 2h A) 1 : 2 : 3 B) 1: 1/2 : 1/3
A) B) Zero C) D) None C) 3 : 2 : 1 D) 1 : 1: 1
 2
36. The energy of H-atom in nth orbit is En then
25. Multiplication of electron velocity and radius
energy in nth orbit of singly ionised helium
for a orbit in an atom is
atom will be
A) Proportional to mass of electron
A) 4En B) En/4 C) 2En D) En/2
B) Proportional to square of mass of electron
C) Inversely proportional to mass of electron
Atomic Structure
37. The energy of second Bohr orbit of the C) Potential energy decree
hydrogen atom is –328 KJ/mol. Hence the D) None of these
energy of fourth Bohr orbit should be. 44. Maximum frequency of emission is obtained
A) – 41 KJ/mol B) – 1312 KJ/mol for the transition.
C) – 164 KJ/mol D) – 82 KJ/mol A) n = 2 to n = 1 B) n = 6 to n = 2
38. In a hydrogen atom, if energy of an electron C) n = 1 to n = 2 D) n = 2 to n = 6
in ground state is –13.6 eV, then engery in 45. If the ionization energy of hydrgoen is 313.8K
the 2nd excited state is cal per mole, then the energy of the electron
A) –1.51 eV B) –3.4 eV in 2nd excited
C) –6.04 eV D) –13.6 eV A) -113.2 Kcal/mole B) -78.45 Kcal/mole
39. The ratio between kinetic energy and the C) -313.8 Kcal/mole D) -35 Kcal/mole
total energy of the electrons of hydrgoen 46. Which of the following electron transition
atom according to Bohr’s model is. will require the largest amount of energy in
A) 2 : 1 B) 1 : 1 C) 1 : –1 D) 1 : 2 a hydrogen atom.
40. Potential energy is –27.2eV in second orbit A) From n = 1 to n =2 B) From n = 2 to n =3
of He+ then calculate, double of total energy C) From n =  to n =1D) From n = 3 to n =5
in first excited state of hydrgen atom 47. If the potential energy (PE) of hydrogen

.in
A) –13.6 eV B) – 54.4 eV electron is –3.02eV then in which of the
C) – 6.8 eV D) – 27.2 eV following excited level is electron present.
41. The energy levels for Z A (  z1) can be given A) 1st B) 2nd C) 3rd D) 4th
al
48. The radiation of low frequency will be
by
emitted in which transition of hydrogen
rn
A) En for A (  z1)  Z2  E n for H atom.
B) En for A (  z1)  Z  E n for H A) n = 1 to n = 4 B) n = 2 to n = 5
ou

C) n = 3 to n = 1 D) n = 5 to n = 2
(  z 1) 1 49. A single electron orbits a stationary nucleus
C) En for A   E n for H
Z2 (z =5) . The energy required to excite the
uj

1
electron from third to fourth Bohr orbit will
(  z 1)
D) En for A 
 E n for H be
Ed

Z
A) 4.5 eV B) 8.53 eV
42. The graphical representation of energy of e– C) 25 eV D) 16.53 eV
and atomic number is 50. The ratio of energies of hydrogen atom for
first and second excited state is
A) 4/1 B) 1/4 C) 4/9 D) 9/4
2
E E 51. En = –313.6/ n . If the value of En = –34.84
A) B) then to which of the following values does
Z2 Z2 ‘n’ correspond.
A) 1 B) 2 C) 3 D) 4
52. The ratio of potential energy and total energy
of an electron in a Bohr orbit of hydrogen
E Z2 like species is.
E
C) D) A) 2 B) –2 C) 1 D) –1
Z2 53. Which is not a correct order of energy for 1,
2nd & 3rd orbit.
43. Going from K-shell to N-shell in case of H- A) E1  E 2  E 3
atom
A) Kinetic energy decreases B) (PE)1  (PE) 2  (PE)3
B) Total energy decreases
Atomic Structure
C) (KE)1  (KE)2  (KE)3 3
A) of its ionisation energy
D) ‘1’ & ‘3’ both 4
54. Which of the following is a correct 1
relationship B) of its ionisation energy
2
A) E1 of H = 1/2 E2 of He+ = 1/3 E3 of Li+2 =
1/4 E3 of Be+3 1
B) E1 (H) = E2 (He+) = E3 (Li+2) = E4 (Be+3) C) of its ionisation energy
4
C) E1 (H) = 2 E2 (He+) = 3E3 (Li+2) = 4 E4 (Be+3) D) None
D) No relation 62. The ionisation potential of a singly ionised
55. Which is correct for any H like species helium ion is equivalent to
A) (E2–E1)>(E3– E2)> (E4–E3) A) Kinetic Energy of first orbit
B) (E2 – E1) < (E3 – E2) < (E4 – E3) B) Energy of last orbit
C) (E2 – E1) = (E3 – E2) = (E4 –E3) C) Average energy in orbits
D) (E2 – E1) = 1/4 (E3 - E2) =1/9 (E4 – E3) D) Maximum energy in orbits
56. Which of the following is a correct graph 63. The ionisation energy for the H- atom is
13.6 eV, ground state to next higher state will be
A)3.4 B)10.2 C)12.1 D)1.5

.in
K.E. K.E.
A) B) SPECTRUM AND SPECTRAL LINES
64. The spectrum of He is expected to be similar
al
n n
to that of
A) H B) Na C) He+ D) Li+
rn

65. Third line of Balmer series is produced by


K.E. K.E. which transition in spectrum of H– atom
ou

C) D) A) 5 to 2 B) 5 to 1 C) 4 to 2 D) 4 to 1
 66. Which one of the following electron
n Z2
uj

transitions between energy levels produces


57. First excitation potential of H atom is the line of shortest wavelength in hydrogen
Ed

A) 10.2 eV B) 3.4 eV spectrum ?


C) 0 D) –3.4 eV A) n2  n1 B)n3  n1
58. Energy required to remove an e– from M shell C) n4  n1 D) n4  n3
of H–atom is 1.51 eV, then energy of first 67. Which series have highest energy in
excited state will be hydrogen spectrum
A) –1.51eV B) +1.51 eV A) Balmer B) Bracket
C) –3.4eV D) –13.6 eV C) Pfund D) Lyman
59. The ionisation potential of the hydrogen 68. The ratio of minimum frequency of Lyman
atom is 13.6 eV. The energy needed to ionise & Balmer series will be
a hydrogen atom which is in its second A) 1.25 B) 0.25 C) 5.4 D) 10
excited state is about 69. Which transition emits photon of maximum
A) 13.6 eV B) 10.2 eV frequency
C) 3.4 eV D) 1.5 eV A) second spectral line of Balmer series
60. The ionisation energy for excited hydrogen B) second spectral line of Paschen series
atom in eV will be C) fifth spectral line of Humphery series
A) 13.6 B) Less than 13.6 D) first spectral line of Lyman series
C) Greater than 13.6 D) 3.4 or less 70. Which one of the following species will give
61. The energy required to excite an electron of a series of spectal lines similar to that of Mg2+.
H- atom from first orbit to second orbit is . A) Al3+ B) Na C) Mg+ D) F
Atomic Structure
71. The ratio of minimum wavelengths of Lyman 81. I n H -atom, electr on tr ansits f r om 6th orbit to
& Balmer series will be. 2nd orbit in multi step. Then total spectral
A)1.25 B)0.25 C)5 D)10 lines (without Balmer series) will be
72. The wavelength of photon obtained by A) 6 B) 10 C) 4 D) 0
electron transition between two levels in H- 82. An atom has x energy level, then total
atom and singly ionised He are 1 and  2 number of lines in its spectrum are.
A) 1+2+3........ (x+1) B) 1+2+3...... (x2)
respectively, then
C) 1+2+3........(x–1) D) (x+1)(x+2)(x+4)
A)  2  1 B)  2  21 83. The figure indicates the energy level diagram
C)  2  1 / 2 D)  2  1 / 4 for the origin of six spectral lines in emission
73. Find out ratio of following for photon spectrum(e.g. line no. 5 arises from the
transition from level B to X) which of the
(v max ) Lyman : (v max ) Brakett following spectral the absorption spectrum.
A) 1 : 16 B) 16 : 1 C) 4 : 1 D) 1 : 4
74. The ratio of wavelengths of first line of C
Lyman series in Li+2 and first line of Lyman B
series in deuterium (1H2) is X
A

.in
A) 1 : 9 B) 9 : 1 C) 1 : 4 D) 4 : 1 1 2 3 4 5 6
75. In an electronic transition atom cannot emit
A) 1, 2, 3 B) 3, 2
A) Visible light B)  - rays
C) 4, 5,6 D) 3, 2, 1
al
C) Infra red light D) Ultra violet light
84. A certain electronic transition from an
76. The first Lyman transition in the hydrogen
rn
excited state to ground state of the H2 atom
spectrum has E = 10.2 eV. The same energy
in one or more step gives rise to three lines
change is observed in the second Balmer in the ultra violet region of the spectrum.
ou

transition How many lines does this transition produce


A) Li2+ B) Li+ C) He+ D) Be3+ in the infrared region of the spectrum
77. The limiting line in Balmer series will have
uj

A) 1 B) 2 C) 3 D) 4
a frequency of 85. Four lowest energy levels of H–atoms are
Ed

A) 3.65  10 4 sec 1 B) 3.29  1015 sec 1 shown in the figure. The number of emission
C) 8.22  1014 sec 1 D) 8.22  1014 sec 1 lines could be.
78. If the shortest wavelength of Lyman series 4
of H atom is x, then the wave length of first 3
line of Balmer series of H atom will be. 2
n=1
9x 36x 5x 5x
A) B) C) D) A) 3 B) 4 C) 5 D) 6
5 5 9 36
79. The first emission line in the H-atom 86. In the above problem, the number of
spectrum in the Balmer series will have wave absorption lines could be.
number A) 3 B) 4 C) 5 D) 6
87. If 9.9eV energy is supplied to H atom, the
5R 3R no.of spectral lines emitted is equal to.
A) cm 1 B) cm 1
36 4 A) 0 B) 1 C) 2 D) 3
7R 9R DE – BROGLIE CONCEPT AND
C) cm 1 D) cm 1
144 400 HEISENBERG PRINCIPLE
80. What transition in He+ will have the same  88. An electron has kinetic energy 2.8  10 23 J .
as the I line in Lyman series of H – atom de-Broglie wavelength will be nearly.
A) 5  3 B) 3  2 C) 6  4 D) 4  2
Atomic Structure
(m e  9.1 1031 kg) 98. Heisenberg Uncertainity principle is not valid
for
A) 9.28  1024 m B) 9.28  107 m A) Moving electron B) Motor car
C) 9.28  108 m D) 9.28  1010 m C) Stationary particles D) 2 & 3 both
89. What is the de–Broglie wavelength 99. What should be the momentum (in gram
associated with the hydrogen electron in its centimetre per second) of a particle if its de-
third orbit. Broglie wavelength is lAo and the value of h
A) 9.96  10 10 cm B) 9.96  10 8 cm is 6.6252  10 27 erg
C) 9.96 104 cm D) 9.96 108 cm A) 6.6252  10–19 gcm/s
B) 6.6252  10–21 gcm/s
90. If the de-Broglie wavelength of the fourth
C) 6.6252  10–24 gcm/s
Bohr obit of hydrogen atom is 4A o, the
D) 6.6252  l0–27gcm/s
circumference of the orbit will be.
100. What should be the mass of the photon of
A) 4Ao B) 4nm C) 16Ao D) 16nm
sodium if its wavelength is 5894A o, the
91. No.of wave in fourth orbit.
A) 4 B) 5 C) 0 D) 1 velocity of light is 3  108 metre / second and
92. What is the ratio of the De-Broglie wave the value of h is 6.6252  1034 kgm 2 / s ?
lengths for electrons accelerated through

.in
A) 3.746  1026 kg B) 3.746  1030 kg
200volts and 50 volts.
A) 1: 2 B) 2 : 1 C) 3 : 10 D) 10: 3 C) 3.746  1034 kg D) 3.746  1036 kg
al
93. For a valid Bohr orbit, its circumfrence 101. Which of the following has least de -Broglie
should be
?
rn
A)  n B)  (n  1) A) e– B) p C) CO2 D) SO2
C)  n D)  n
ou

QUANTUM NUMBERS
64. A particle X moving with a certain velocity
has a debroglie wavelength of 1Ao. If particle 102. The following quantum no. are possible for
how many orbitals n =3,  =2, m = +2
uj

Y has a mass of 25% that of X and velocity


75% that of X, debroglies wavelength of Y A) 1 B) 2 C) 3 D) 4
103. Number of possible orbitals (all types) in n
Ed

will be.
A) 3Ao B) 5.33Ao C) 6.88Ao D) 48Ao =3 energy level is
95. The number of waves made by a Bohr A) 1 B) 3 C) 4 D) 9
electron in an orbit of maximum magnetic 104. Which sub-shell is not permissible.
quantum number +2. A) 2d B) 4f C) 6p D) 3s
A) 3 B) 4 C) 2 D) 1 105. Nodal plane is found in which orbital.
96. The uncertainity in position of an electron & A) n = 2,  = 0 B) n = 3,  = 0
helium atom are same. If the uncertainity in C) n =2,  = 1 D) n = 1,  = 0
momentum for the electron is 32  105, then 106. No. of nodal surface in 2s orbital
the uncertainity in momentum of helium A) 0 B) 1 C) 2 D) 3
atom will be 107. Number of orbitals in h sub-shell is
A) 32  l05 B) 16  105 A) 11 B) 15 C) 17 D) 19
C) 8  105 D) None 108. How many quantum numbers are required
97. The uncertainty in the position of an electron to
(mass 9.1 1028 gm ) moving with a velocity specify the position of electron
A) 1 B) 2 C) 3 D) 4
of 3  104 cm sec 1 , uncertainity in velocity is 109. Which of the following is correct for a 4d-
0.011% will be. electron
A) 1.92 cm B) 7.68 cm
C) 0.175 cm D) 3.84 cm
Atomic Structure
1 121 Any nf-orbital can accomodate upto
A) n  4,   2,s   A) 14 electron
2
B) Six electrons
B) n  4,   2,s  0 C) Two electrons with parallel spin
C) n  4,   3,s  0 D) Two electrons with o ppositespin
122 n,  and m values of an electron in 3py orbital
1
D) n  4,   3,s   are
2 A) n = 3 ;  = 1 and m = 1
110. If n = 3, then which value of  is correct B) n = 3 ;  = 1 and m = –1
A) 0 B)1 C) Both 1 and 2 are correct
C)2 D) All of them D) None of these
111. Energy of atomic orbitals in a particular shell 123. 36Kr has the electronic configuration (18Ar)
is in order. 4s2 3d10 4p6. The 39th electron will go into
A) s<p<d<f B) s>p>d>f which one of the following sub-levels
C) p<d<f<s D) f >d>s>p A) 4f B) 4d C) 3p D) 5s
112. Which statement is not correct for n = 5, m = 2 124 The maximum probability of finding an
A)  = 4 B)  = 0, 1,2,3 ;s = + 1/2 electron in the dxy orbital is

.in
C)  = 3 D)  = 2, 3, 4 A) Along the x-axis
113. Spin angular momentum for electron B) Along the y-axis
h h C) At an angle of 45° from the x and y axis
al
A) s(s  1) B) 2s(s  1) D) At an angle of 90° from the x and y axis
2 2
125. Which orbitlal has two angular
rn
h nodal planes
C) s(s  2) D) None
2 A) s B) p C) d D) f
ou

114. The maximum number of electrons in a p- 126. An orbital with  = 0 is symmetrical about
orbital with n = 6 and m = 0 can be the
A) 14 B) 6 C) 2 D) 10 A) x-axis only B) y-axis only
uj

115. The total value of m for the electrons (n - 4) C) z-axis only D) The nucleus
is
Ed

127. If n &  are principal and azimuthal


A) 4 B) 8 C) 16 D) 32 quantum no. respectively then the expression
116 In an atom, for how many electrons, the for calculating thetotal no. of electron in any
quantum numbers will be, n = 3, l =2, m = +2, energy level is
s= +1/2 n   n 1
A) 18 B) 6 C) 24 D) 1 A)  2(2  1) B)  2(2  1)
117 Which orbital is represented by the complete  0  1

wave function  420   n 1   n 1

A) 4d B) 3d C) 4p D) 4s C) 
 0
2(2  1) D)  2(2  1)
 0
118 An electron is in one of 4d orbital. Which of
the following orbital quantum number value RULES FOR FILLING OF ORBITALS
is not possible 128. Which configuration does not obey pauli’s
A) n=4 B)   1 C) m = 1 D) m = 2 exclusion principle.
119 A neutral atom of an element has 2K.8L, 11M
A)  
and 2N electrons. The number of s-electron
in the atom are B)    
A) 2 B) 8 C) 10 D) 6
120 If   3 then type and number of orbital is C)   
A) 3p, 3 B) 4f, 14 C) 5f, 7 D) 3d, 5
Atomic Structure
A) 3px  3s B) 3dxy  3dyz
D)    
C) 3s  3dxy D) All the above
129. Which of the following configuration follows 137. In ground state of Cr24, number of orbitals
the Hund’s rule with paired and unpaired electron.
2p A) 10 B) 12 C) 15 D) 18
2s
A) [He]    138. For Na (Z = 11) set of quantum numbers for
last electron is
2s 2p 1
B) [He]    A) n  3,   1, m  1,s  
 2

2p 1
2s B) n  3,   0, m  0,s  
C) [He]  2
  
1
2s 2p C) n  3,   0, m  1,s  
2
D) [He] 
 
1
130. The basis of three unpaired electrons present D) n  3,   1, m  1,s  
2

.in
in the configuration of nitrogen is 139. Which of the following set of quantum
A) Aufbau principle B) Pauli’s principle numbers is correct for the 19th electron of
al
C) Hund’s principle D)Uncertainty principle Chromium
131. The orbital with maximum energy is n  m s
rn
A) 3d B) 5p C) 4s D) 6d A) 3 0 0 1/2
132. n and  values of an orbital ‘A’ are 3 and 2, B) 3 2 –2 1/2
ou

of another orbital ‘B’ are 5 and 0. The energy C) 4 0 0 1/2


of D) 4 1 –1 1/2
A) B is more than A 140. Which set of quantum number is correct for
uj

B) A is more than B an electron in 3p orbital


Ed

C) A and B are of same energy 1


D) None A) n  3,   2, m  0,s  
2
133. No. of all subshells of n +  = 7 is 1
A) 4 B) 5 C) 6 D)7 B) n  3,   0, m  1,s  
2
134. Electronic configuration
1
C) n  3,   1, m  1,s  
2
has violated. 1
A) Hund’s rule B) Pauli’s principle D) n  3,   1, m  0,s  
2
D) (n  ) rule
C) Aufbau principle 141. An atom of Cr [Z = 24] loses 2 electrons. How
135. The total spin resulting from a d 9 many unpaired electrons shall be there in
configuration is Cr+2
A) 4 B) 3 C) 2 D) 1
1 3
A) B) 2 C) 1 D) 142. The atomic weight of an element is double
2 2 its atomic number. If there are three elecrons
136. Which of the following transition neither in 2p sub-shell, the element is.
shows absorption nor emission of energy in A) C B) N C) O D) Ca
case of Hydrogen atom
Atomic Structure
143. The atomic number of an element is 17, the is 58 a.m.u. The number of neutrons in its
number of orbitals containing electron pairs nucleus would be
in the A)30 B)32 C)34 D)42
A) 8 B)2 C)1 D)6 152. In an atom having 2K, 8L, 8M and 2N
144. A transition metal ‘X’ has a configuration electrons, the number of electrons with m =
[Ar] 3d5 b its + 3 oxidation state. Its atomic 1
number is 0; S =  aree
2
A) 22 B)26 C)28 D)19
2 A) 6 B) 2 C) 8 D) 16
145. 4s is the configuration of the outermost orbit
153. The number of electrons in the M-shell of
of an element. Its atomic number would be
the element with atomic number 24 is
A)29 B)24 C)30 D)19
A) 24 B)12 C) 8 D)13
146. Sum of the paired electrons present in the
orbital with  = 2 in all the species Fe2+, Co2+
and Ni+2 are
A)9 B)12 C)6 D)15
1) C 2) A 3) A 4) C 5) B
147. What is the electronic configuration of an
6) B 7) D 8) A 9) C 10) B
element in its first excited state which is
11) B 12) A 13) C 14) D 15) C

.in
isoelectronic with O2
16) C 17) C 18) A 19) C 20) C
A) [Ne] 3s2 3p3 3d1 B) [Ne] 3s2 3p4
21) D 22) C 23) D 24) A 25) C
C) [Ne] 3s’ 3p3 3d2 D) [Ne] 3s1 3p5
al
26) A 27) A 28) C 29) B 30) D
148. The quantum number of 20th electron of
31) D 32) D 33) A 34) A 35) B
Fe(Z = 26) ion would be
rn
36) A 37) D 38) A 39) C 40) C
A) 3,2,–2, –1/2 B) 3,2,0,1/2
1 41) A 42) D 43) A 44) A 45) D
C) 4,0,0,+ /2 D) 4,1,–1, +1/2
ou

46) A 47) B 48) D 49) C 50) D


149. The atomic number of the element having
51) C 52) A 53) A 54) B 55) A
maximum number of unpaired 3p electrons
56) C 57) A 58) C 59) D 60) D
is (in ground state)
uj

61) A 62) A 63) B 64) D 65) A


A) 15 B) 10 C) 12 D) 8
66) C 67) D 68) C 69) D 70) A
150. Which one represent is in ground state
Ed

71) B 72) D 73) B 74) A 75) B


configuration
76) C 77) C 78) B 79) A 80) D
81) A 82) C 83) C 84) A 85) D
86) A 87) A 88) C 89) B 90) C
A)
91) A 92) A 93) A 94) B 95) A
96) A 97) C 98) D 99) A 100) D
101) D 102) A 103) D 104) A 105) C
106) B 107) A 108) C 109) A 110) D
B)
111) A 112) B 113) A 114) C 115) C
116) D 117) A 118) B 119) B 120) C
121) D 122) C 123) B 124) C 125) C
126) D 127) D 128) B 129) A 130) C
C)
131) D 132) A 133) A 134) A 135) A
136) D 137) C 138) B 139) C 140) D
141) A 142) B 143) C 144) B 145) C
D) 146) B 147) A 148) C 149) A 150) C
151) B 152) A 153) D
151. The electronic configuration of a dipositive
metal ion M2+ is 2, 8, 14 and its ionic weight
Atomic Structure
1
95. 
v
14. Let percentage of first isotpe be x then average  200 50 1 1
  
20  x  22 100  x 50 200 4 2
at. wt   20.2
100 134. n    7
20 x  2200  22 x  2020
2 x  180 n 
Isotope 1  90%, 2  10% 7 0  7s
6 1 6p
36. For He  , Z  2
5 2  5d
1
En  13.6  (for H atom) 4 3 4 f
n2
3 4  can not be  n
4
En  13.6  2 (for He+ ion)
n

.in
68. Minimum frequency of spectral line is for first
line of psectral series
al
 1 1
RZ 2    AIPMT 2008
vmin  lyman  1 4  27
rn
  1. If uncertainty in position and momentum are
vmin  Balmer  21 1 5
RZ    equal, then uncertainty in velocity is ?
 4 9
ou

h 1 h h 1 h
74. vmax lyman  n   to 1 A) B) C) D)
 2m  2 m 
uj

vmax Bracket  n   to 4 2. The measurement of the electron position is


use Rydberg formulae associated with an uncertainty in
Ed

79. Minimum wavelength is for last line of spectral momentum, which is equal to 1 1018 g cm
series. s–1. the uncertainty in electron velocity is
1 1 (mass of electron = 9  10 28 g)
 min   x
1 1  R A) 1 1011 cm s 1 B) 1 109 cm s 1
R.12   
1 
C) 1 106 cm s 1 D) 1 105 cm s 1
1 36 36 x
1  Balmer     AIPMT 2009
21 1  5R 5
R.1    3. Maximum number of electrons in a subshell
 4 9
atom is determined by the following
1 A) 2n2 B) 4  +2 C) 2  + 1 D) 4  – 2
90. v3  2.18  106  m / s  7.27  105 m / s
3 4. Which of the following is not permissible
arrangement of electrons in an atom ?
h 6.626  1027
  A) n = 3,  = 2, m = –2, s = –1/2
mv 9.1  1028  7.27  108
B) n = 4,  = 0, m = 0, s = –1/2
 9.96  108
C) n = 5,  = 3, m = 0, s = +1/2
D) n = 3,  = 2, m = –3, s = –1/2
Atomic Structure
AIPMT 2010 AIPMT PRE 2012
5. A 0.66 kg ball is moving with a speed of 100 12. Maximum number of electrons in a subshell
m/s. The associated wavelength will be (h = with  =3 and n = 4 is.
6.6  10 34 Js) A) 10 B) 12 C) 14 D) 16
A) 6.6  10 34 m B) 1.0 1035 m 13. The correct set of four quantum numbers for
the valence electron of rubidium atom (Z =
C) 1.0  1032 m D) 6.6  10 32 m 37) is
1
AIPMT Pre – 2011 A) 5, 0, 0, + 2 B) 5, 1, 0, + 1 2
6. The total number of atomic orbitals in fourth 1 1
C) 5, 1, 1, + 2 D) 6, 0, 0 + 2
energy level of an atom is
A) 8 B) 16 C) 32 D) 4 AIPMT MAINS 2012
7. The energies E1 and E2 of two radiations 14. The orbital angular momentum of a p-
are 25eV and 50eV respectively. The electron is given as
relation between their wavelengths i.e.
3h h h h
1 and  2 will be A) B) 6 C) D) 3
2 2 2 2

.in
A) 1   2 B) 1  2 2
AIIMS 2012
1
al
C) 1  4 2 D) 1   2 15. Threshold frequency of a metal is 5  1013 sec–1
2
upon which 1 1014 sec–1 frequency light is
8. If n = 6, the correct sequence for filling of
rn
focused then maximum kinetic energy of
electrons will be emitted electron.
A) ns  (n  2)f  (n  1)d  np
ou

A) 3.3  1021 B) 3.3  1020


B) ns  (n  1)d  (n  2)f  np C) 6.6  1021 D) 6.6  10 20
C) ns  (n  2)f  np  (n  1)d
uj

nh
D) ns  np(n  1)d  (n  2)f 16. In Bohr’s orbit indicates.
2
Ed

AIPMT Mains 2011 A) Momentum B) Kinetic energy


9. According to the Bohr Theory, which of the C) Potential energy D) Angular momentum
following transitions in the hydrogen atom NEET UG 2013
will give rise to the least energetic photon ?
17. The value of Planck’s constant is 6.63 1034 Js
A) n = 5 to n = 3 B) n = 6 to n - 1
C) n = 5 to n = 4 D) n = 6 to n = 5 The speed of light is 3  1017 nm s–1. Which valu
is closest to the wavelength in nanometer of i
AIIMS - 2011
quantum of light with frequency of 6  1015 s–1
10. Smallest wavelength occurs for
A) 75 B)10 C) 25 D) 50
A) Lyman series B) Balmen series
2
C) Paschen series D) Bracket series 18  Z 
18. Based on equation E  2.178  10 J  2
11. Which of the following is wrong for Bohr n 
model certain conclusions are written. Which of
A) It establishes stability of atom them is not correct ?
B) It is contradicted with Heisenberg A) For n=1, the electron has a more negative
uncertainity principle energy than it does for n=6 which means that
C) It explain the concept of spectral lines the elecron is more lossely bound in the
D) e– behaves as particle & wave smallest allowed orbit.
Atomic Structure
B) The negative sign in equation simply means A) ClO3 ,CO32 B) SO32 , NO3
that the energy of electron bound to the nucleus
is lower than it would be if the electrons were C) ClO3 ,SO32 D) CO32 ,SO32
at the infinite distance from the nucleus 26. The number of d-electrons in Fe2+(Z=26) is
C) Larger the value of n, the larger is the orbit not equal to the number of electrons in which
radius one of the following ?
D) Equation can be used to calculate the change A) p - electrons in Cl (Z=17)
in energy when the electron change orbit. B) d - electrons in Fe (Z=26)
19. What is the miaximum numbers of electrons C) p - electrons in Ne(Z=10)
that can be associated with the following set D) s - electrons in Mg(Z=12)
of quantum numbers ? n=3, l = 1 and m = –1 27. Magnetic moment 2.84 B.M. is given by
A) 2 B) 10 C) 6 D) 4 (At.no.), Ni = 28, Ti = 22, Cr = 24, Co = 27)
AIIMS 2013 A) Ti3+ B) Cr2+ C) Co2+ D) Ni2+
28. The angular momentum of electron in ‘d’
20. A particle is moving with 3 times faster than
orbital is equal to
speed of e–. Ratio of wavelength of particle
& electron is 1.8  104 then particle is A) 2h B) 2 3h C) 0 h D) 6h

.in
A) Neutron B)  -particle RE – AIPMT 2015
C) Deutron D) Tritium 29. Which is the correct order of increasing
al
AIPMT 2014 energy of the listed orbitals in the atom of
titanium ?
rn
21. What is the maximum number of orbitals
that can be identified with the following ( At. no. Z =22)
quantum numbers ? A) 3s 3p 3d 4s B) 3s 3p 4s 3d
ou

C) 3s 4s 3p 3d D) 4s 3s 3p 3d
n = 3,  = 1, m  0
A) 1 B) 2 C) 3 D) 4 AIIMS 2015
uj

22. Calculate the energy in joule corresponding to light 30. In which transition of hydrogen atom have
of wavelength 45nm: (Planck’s constant same wavelength as in Balmer series
Ed

–1 transition of He+ ion (n = 4 to n = 2)


h  6.63  10 34 Js; speed of light c  3  108 ms )
A) 4 to 2 B) 3 to 2 C) 2 to 1 D) 4 to 1
A) 6.67  1015 B) 6.67  1011
NEET-I 2016
C) 4.42  1015 D) 4.42  1018
31. Two electrons occupying the same orbital are
23. Magnetic moment 2.83 BM is given by which
distinguished by
of the following ions ?
A) aximuthal quantum number
(At. nos. Ti = 22, Cr = 24, Mn = 25, Ni = 28):
B) spin quantum number
A) Ti3+ B) Ni2+ C) Cr3+ D) Mn2+
C) principal quantum number
AIIMS 2014 D) magnetic quantum number
24. The energy of an electron of 2py orbital is
NEET-II 2016
A) greater than 2px orbital
32. Which of the following pairs of d-orbitals will
B) Less than 2pz orbial
have electron density along the axes ?
C) same as that of 2px and 2pz orbital
D) Equal to 2s orbital A) d z 2 , d xz B) d xz , d yz

AIPMT 2015 C) d z 2 , d x2  y 2 D) d xy , d x2  y 2
25. Which of the following pairs of ions are
isoelectronic and isostructural ?
Atomic Structure
33. How many electrons can fit in the orbital for h h
which n=3 and l=1 ? x.P  P.P 
4 4
A) 2 B) 6 C) 10 D) 14
h 1 h
NEET 2017  mV  2   V 
34. Which one is the wrong statement ?
4 2m 
A) The uncertainty principle is 1
7. E
h 
E  t 
4 E1  2 25  2
    1  2 2
B) Half filled and fully filled orbitals have E2 1 50 1
greater stability die to greater exchange energy,
greater symmetry and more balanced 30. Given :  H   He ,
arrangement.
1 1
C) The energy of 2s-orbital is less than the enery 
  1 1 
of 2p-orbital in case of hydrogen like atoms. or R.11 12  12 R.4   
 n n   4 16 
D) de-Broglie’s wavelength is given by 1 2

.in
h 1 1 3
 , where m = mass of the particle, v =  
mv n12 n22 4
group velocity of the particle.
al
n1  1 and n2  2
NEET 2018
rn
31. For the two electrons occupying the same
35. Which one is the wrong statement ?
orbital values of n, l and ml are same but ms is
A) Total orbital angular momentum of electron
different, i.e.+ 1/2 and - 1/2.
ou

in ‘s’ orbital is equal to zero


B) An orbital is designated by three quantum 32. d x2  y 2 and d 2 orbitals have electron density
z
numbers while an electron in an atom is
uj

along the axes while dxy and dyz and dxz orbitals
designated by four quantum numbers. have electron density inbetween the axes.
C) The electron configuration of N atom is
Ed

33. For n=3 and l =1, the subshell is 3p and a


1s 2 2s 2 2p1x 2 p1y 2 p1z particular 3p orbital can accommodate only 2
electrons.
     34. In case of hydrogen like atoms, energy depends
on the principal quantum number only. Hence,
D) The value of m for d z 2 is zero 2s-orbital will have energy equal to 2p-orbital.
35. The correct configuration of ‘N’ is

1) B 2) B 3) B 4) D 5) B 6) B 7) B
8) A 9) D 10) A 11) D 12) C 13) A 14) C
15) B 16) D 17) D 18) A 19) B 20) A 21) A
22) D 23) B 24) C 25) C 26) A 27) D 28) D
29) B 30) C 31) B 32) C 33) A 34) C 35) 3 1. Which of the following has maximum energy.
3s 3p 3d
A)

3s 3p 3d
1. x  P    mv   mV B)
Atomic Structure
3p 11. The quantum numbers +1/2 and –1/2 for the
3s 3d
C) electron spin represent.
A) Rotation of the electron in clockwise and
3s 3p 3d anticlockwise direction respectively.
D) B) Rotation of the electron in anticlockwise and
clockwise direction respectively.
2. The frequency of radiation emitted when the C) Magnetic moment of the electron pointing
electron falls from n = 4 to n = 1 in a hydrogen up and down respectively.
atom will be (Given ionization energy , D) Two quantum mechanical spin states which
–1
H  2.15  1018 Jatom ) have no classical analogue.
A) 1.03  1015 s 1 B) 3.08  1015 s 1 12. Uncertainty in position of a 0.25g particle is
C) 2.00  1015 s 1 D) 1.54  1015 s 1 10–5m. Then uncertainty in its velocity will be
3. Which orbital diagram does not obey Aufbau (h  6.6  1034 Js)
principle. A) 1.2 1034 B) 2.11026
A) B) C) 1.6 1020 D) 1.7  109

.in
13. The wavelength of radiation emitted when
C) D) an electron in a hydrogen atom makes a
4. Quantum number which can not be transition from an energy level with n=3 to a
al
calculated by schrodinger wave equation is level with n=2 is [Given that
A) n B)  C) m D) s 1312
rn
En  kJ mol–1]
h n2
5. is angular momentum in .......orbit of He+
ou

2 A) 6.56  107 m B) 65.6 m


A) First B) Second C) 65.6  107 m D) any of the above
C) Third D) Infinite
uj

14. Electronic energy of hydrogen atom depends


6. Which element contain non spherical on the quantum number
electron density.
Ed

A) n,  and m B) n and  only


A) He B) B C) Be D) Li
+
C) n and m only D) n only
7. First shell energy of He ion is –54.4 eV. Then 15. The energy required to escape the electron
energy of its second shell is. from ground state of H is 13.6 eV then the
A) –54.4 eV B) –13.6 eV same for Ist excited state of H atom .
C) –27.2 eV D) +27.2eV A) 3.4 B) 13.6 C) 27.2
8. If  =2 and n = 3 for any atom then maximum D) can’t say anything
number of electron is 16. A gas absorbs a photon of 355nm and emits
A) 2 B) 6 C) 12 D) 10 at two wavelengths. If one of the emissions is
9. A metal in its dipositive state has the at 680 nm, the other is at
electronic configuration 2,8,14 and has the A) 743 nm B) 518 nm
atomic weight equal to 56. Number of C) 1035 nm D) 325 nm
neutrons in its nucleus would be 17. The frequency of light emitted for the
A) 30 B) 32 C) 34 D) 28 transition n = 4 to n = 2 of He+ is equal to the
10. What is the maximum number of electrons transition in H atom corresponding to which
which can be accoummodated in an atom in of the following
which the highest principal quantum number A) n = 3 to n = 1 B) n = 2 to n = 1
value is 4 C) n = 3 to n = 2 D) n = 4 to n = 3
A) 10 B) 18 C) 36 D) 54
Atomic Structure
18. The electron identified by quantum numbers A) 1.89 eV B) 2.55 eV
n and  . C) 12.09 eV D) 12.75 eV
a) n = 4,  = 1 b) n = 4,  = 0 26. If a photon of energy 14 eV. is incident on an
H-atom, what is true
c) n = 3,  = 2 d) n = 3,  = 1
A) Atom will be ionised and electron will
Can be placed in order of increasing energy
as have a kinetic energy of 14 eV
A) (a) < (c) < (b) < (d) B) (c) < (d) < (b) < (a) B) Atom will be ionised and electron will have
C) (d) < (b) < (c) < (a) D) (b) < (d) < (a) < (c) a kinetic energy of 0.4 eV
19. Energy of an electron is given by C) Photon passes through atom without
interacting with it
18  Z2  D) More than one electrons will make
E  2.178 10 J  2  . Wavelength of
n  transitions
light required to excite an electron in an 27. An electron of energy 10.8 eV is incident on
hydrogen atom from level n = 1 to n = 2 will an H– atom then
be. A) The electron will come out with 10.8 eV
energy.
( h  6.62  10 34 Js and c  3.0  108 ms–1)
B) The electron will be completely absorbed

.in
A) 1.214  107 m B) 2.816  10 7 m C) 10.2 eV. of the electron would be absorbed
C) 6.500  107 m D) 8.500  107 m by H atom and it would come out with 0.6 eV
al
20. A certain negative ion X–2 has in its nucleus energy.
18 neutrons and 18 electrons in its extra D) None
rn
nuclear structure. What is the mass number 28. The ratio of the difference in energy between
of the most abundant isotope of ‘X’ the first and second Bohr orbit to that
ou

A) 35.46 B) 32 C) 36 D) 39 between second and third Bohr orbit in H-


21. Supposing the I.P. of hydrogen atom is 960 atom is.
A) 4/9 B) 1/3 C) 27/5 D) 1/2
uj

eV. Find out the value of principal quantum


number having the energy equal to –60eV. 29. Match the following
A) Energy of ground state
Ed

A) n = 2 B) n = 3 C) n = 4 D) n = 5
22. If the ionisation potential of an atom is 20V, of He+ i) +6.04 eV
its first excitation potential will be B) Potential energy of I ii) –27.2 eV
A) 5 V B) 10 V C) 15 V D) 20 V orbit of H–atom
23. A single electron orbits a stationary nucleus C) Kinetic energy of II iii) 8.72  10 18 J
of charge +Ze, where Z is a constant. It excited state of He+
requires 47.2 eV to excite electron from D) Ionisation potential of He+ iv) –54.4 eV
second Bohr orbit to third Bohr orbit , find A) A – i, B – ii, C – iii, D – iv
the value of Z. B) A – iv, B – iii, C – ii, D – i
A) 1 B) 3 C) 5 D) 4 C) A – iv, B – ii, C – i, D – iii
24. A photon of energy 12.75 eV is completely D) A – ii, B – iii, C – i, D – iv
absorbed by a hydrogen atom initially in 30. In the following transition which statement
ground state. The principle quantum number is correct
of the excited state is
A) 1 B) 3 C) 4 D)  A) E 31  E 32  E 21 B) 3  1   2
25. An hydrogen atom (ionisation energy 13.6eV) C) v3  v 2  v1 D) All of these
jumps from third excited state to first excited
31. In which transition , one quantum of energy
state. The energy of photon emitted in the
is emmited
process is
Atomic Structure
A) n = 4  n = 2 B) n = 3  n = 1 39. For the azimuthal quantum number (  ), the
C) n = 4  n = 1 D) All of them total number of magnetic quantum nmber is
32. When a hydrogen sample in ground state is given by.
bombarded then what potential is required  m  1  m  1
to accelerate electron so that first Paschen A)   B)  
2 2
line is emitted.
A) 2.55 V B) 0.65 V C) 12.09 V D) 12.75 V  2m  1  2m  1
C)   D)  
33. The binding energy of e– in ground state of 2 2
hydrogen atom is 13.6 eV. The energies 40. In P-atom find out the no. of paired electrons
required to eject out an electron from three for   1 and m = 0
lowest states of He+ atom will be (in eV) A) 3 B) 1 C) 2 D) 0
A) 13.6, 10.2, 3.4 B) 13.6, 3.4, 1.5 41. When the value of principal quantum
C) 13.6, 27.2, 40.8 D) 54.4, 13.6, 6 number n is 3, the permitted value of
34. The transition of electron in H–atom that will azimuthal quantum numbers  and
emit maximum energy is. magnetic quantum numbers ‘m’ are
A) n 3  n 2 B) n 4  n 3 A)  m

.in
0 0
C) n 5  n 4 D) All have same 1 +1,0,–1
energy 2 +2,+1,0,–1,–2
al
35. Given that in the H- atom the transition B)  m
rn
energy for n = 1 to n = 2, Rydberg states is 0 1
10.2eV. The energy for the same transition 2 +2,1,–2
in Be3+ is
ou

3 +3,+3,+2,1,–2,–3
A) 20.4 eV B) 163.2 eV C)  m
C) 30.6 eV D) 40.8 eV 0 0
uj

36. When a electron in H– atom jumps from n = 1 1 2 3 2 0, 1, 2


4 to n = 1, ultra violet light is emitted, if the 2 +3,+2, 1, –2, –3
Ed

transition corresponds to n = 4 to n = 2, which D) m



of the following colours will be emitted 1 0,1
A) Ultra violet B) Green 2 0,1,2
C) Infra red D) No colour 3 0,1,2,3
37. The wavelength of first line of the Lymen 42. A filled or half - filled set of p or d orbitals is
series for hydrogen is 1216 A°. The spherically symmetric. Point out the species
wavelength for the first line of this series for which has spherical symmetry.
a 10 time ionised sodium atom (z= 11) will be A) Na B) C C) Cl– D) Fe
A) 1000 A° B) 100 A° 43. Remaining part of atom except outer orbit is
C) 10 A° D) 1 A° called
38. A certain electronic transition from an A) Kernel B) Core
excited state to the ground state of the H atom C) Empty space D) None of these
in one or more steps gives rise to four lines 44. For H atom, the energy required for the re-
in the ultra violet region of the spectrum, how moval of electron from various sub-shells is
many lines does this transition produce in given as under
the infrared region of the spectrum. The order of the energies would be.
A) 1 B) 2 C) 3 D) 4 A) E1  E 2  E 3 B) E 3  E 2  E1
Atomic Structure
C) E1  E 2  E 3 D) None of these 1 1
45. Radius of H-atom in its ground state is
27.  
E3  E2  13.6 Z 2   
4 9
5.3  1011 m. After collision with an electron
1 1
it is found to have a radius of 21.2  1011 m. 47.2  13.6  Z 2   
4 5
What is the principal quantum number ‘n’
of the final state of the atom. Z 2  25  Z  5
A) n = 2 B) n = 3 C) n = 4 D) n = 16

1) B 2) B 3) B 4) D 5) A 6) B 7) B Directions for Assertion & Reason


8) D 9) A 10) C 11) D 12) B 13) A 14) D questions
15) A 16) A 17) B 18) C 19) A 20) B 21) D These questions consist of two statements each,
22) C 23) C 24) C 25) B 26) B 27) C 28) C printed as Assertion and reason. While
answering these Questions you are required to
29) C 30) C 31) D 32) D 33) D 34) A 35) B
choose any one of the following four responses.
36) B 37) C 38) C 39) B 40) C 41) A 42) C

.in
A) If both Assertion & Reason are True & the
43) A 44) C 45) A
Reason is a correct explanation of the Assertion.
B) If both Assertion & Reason are tru but
al
Reason is not a correct explanation of Assertion
C) If Assertion is True but the Reason is False.
rn

1 2
 1 1   1  1  D) If both Assertion & Reason are false.
16.  R 1  2  2   R     1. Assertion : In Rutherford’s gold foil
ou

   2  3   4  9 
experiment,
very few  – particles are deflected back.
 9  4  5
uj

 R   R  Reason : Nucleus present inside the atom is


 36   36 
heavy.
Ed

36 1  36  A) A B) B C)C D) D
     9.12 106  cm  6.56 107 2. Assertion : Mass number of most of the
5 R 5 
elements are fractional.
2.178  1018
22. E2  E1   2.178  1018
4 Reason : Mass numbers are obtained by
comparing with then mass number of carbon
3
  2.178  10 18 J taken
4 A) A B) B C)C D) D
hc 6.62  1034  3  108 3. Assertion: In an atom, the velocity of electron
  in the higher orbits keeps on decreasing.
E2  E1 3
 2.178  1018 Reason : Velocity of electron is inversely
4
proportional to radius of the orbit.
 1.22  108 m A) A B) B C) C D) D
IP 960 4. Assertion: Total energy of electron in hydrogen
25. E 2
 60  2 atom is negative
n n
Reason : It is in bound state.
960 A) A B) B C) C D) D
n2   16  n  4
60 5. Assertion: Limiting line in the balmer series
has a wavelength of 364.7 nm.
Atomic Structure
Reason : Limiting line is obtained for a jump 14. Assertion : No two electrons in an atom can
of electron from n =  to n = 2 for Balmer have the same values of four quantum numbers.
series. Reason : No two electrons in an atom can be
A) A B) B C) C D) D simulaneously in the same shell, same subshell,
6. Assertion : A spectral line will be seen for a same orbitals and have same spin.
2px – 2py transition. A) A B) B C) C D) D
Reason : Only balmer lines are observed in the 15. Assertion: An orbital cannot have more than
visible region. two electrons
7. Assertion : Bohr model is not suitable in case Reason: The two electrons with opposite spin
of multielectron species. in an orbital create opposite magnetic field.
Reason : It does not tells about electron-electron A) A B) B C) C D) D
16. Assertion: In hydrogen energy of 4s is more
interaction.
than 3d.
A) A B) B C) C D) D
Reason: An orbital with lower value of (n  )
8. Assertion : p–orbit, has dumb-bell shape.
has smaller energy than the orbital with higher
Reason : Electrons present in p – orbital can
have one value of (n  ) .

.in
A) A B) B C) C D) D A) A B) B C) C D) D
9. Assertion : Nodal plane of px atomic orbital is 17. Assertion : The configuration of B atom cannot
be 1s2 2s3
al
yz plane
Reason : Hund’s rule demands that the
Reason : In px atomic orbital, electron density
rn
configuration should display maximum
is zero in the yz plan
multiplicity.
A) A B) B C) C D) D
A) A B) B C) C D) D
ou

10. Assertion : 2p orbitals do not have spherical 18. Assertion : The electronic configuration of
nodes. nitrogen atom is represented as
uj

Reason : The number of spherical nodes in p- Reason : The configuration of ground state of
orbitals is given by (n–2) an atom is the one which has the greates
Ed

A) A B) B C) C D) D multiplicity.
11. Assertion : There are two spherical nodes in 3s A) A B) B C) C D) D
– orbital. 19. Assertion : The ground state configuration of
Reason : There is no angular node in 3s – Cr is 3d5 4s1
orbital. Reason : A set of exactly half filled orbitals
A) A B) B C) C D) D containing parallel spin arrangement provide
12. Assertion : Number of radial and angular node extra stability.
for 3p – orbital are 1, 1 respectively. A) A B) B C) C D) D
+2
Reason : No. of radial and angular node 20. Assertion : M (at.no.25) has magnetic
depends only on principal quantum number. moment (spin only) 5.85 in water.
Reason : Water is weak ligand.
A) A B) B C) C D) D A) A B) B C) C D) D
13. Assertion : For hydrogen 2s & 2p have same
energy.
Reason : For an atom of same principal
1) B 2) D 3) C 4) A 5) A 6) D 7) A
quantum number. s, p, d & f have same energy.
8) B 9) A 10) A 11) B 12) C 13) C 14) A
A) A B) B C) C D) D
15) A 16) B 17) B 18) A 19) A 20) A

S-ar putea să vă placă și